Practice NCLEX 1 Maternal & Child

Ace your homework & exams now with Quizwiz!

A client scheduled for elective cesarean birth in 4 hours asks for a sip of coffee with creamer. How should the nurse respond? 1. "You can drink black coffee." 2. "You may have coffee with creamer." 3. "You are only allowed to drink water right now." 4. "Since you are having surgery today, you're not allowed to have anything to eat or drink."

1

A young adolescent is in active labor but did not know that she was pregnant. What is the most important nursing action? 1. Assess blood pressure and check for proteinuria. 2. Obtain a Social Services referral to discuss adoption. 3. Determine who might be the father of the baby for paternity testing. 4. Ask the client what kind of birthing experience she would like to have.

1

The baseline fetal heart rate is 135 beats per minute. Following contractions, the fetus develops late decelerations. Which nursing intervention should be implemented first? 1. Facilitate a maternal left lateral position. 2. Alert the healthcare provider of the fetal status. 3. Decrease the rate of infusion of intravenous fluids. 4. Administer oxygen to the client at 4 L per minute via nasal cannula.

1

The client in labor arrives at the birthing unit with her partner. Which step of the admission process should be completed first? 1. Welcoming the couple 2. The sterile vaginal examination 3. Auscultation of the fetal heart rate 4. Checking for ruptured membranes

1

The client in labor with meconium-stained amniotic fluid asks why the fetal monitor is necessary because the belt is uncomfortable. What should the nurse explain about monitoring? 1. "It helps us to see how the baby is tolerating labor." 2. "It can be removed, and oxygen can be given instead." 3. "It is necessary so we can see how your labor is progressing." 4. "It will prevent complications from the meconium in your fluid."

1

The nurse encounters a woman giving birth at the local shopping mall. What should the nurse do first? 1. Visualize the perineum. 2. Apply counterpressure to the perineum. 3. Ask a bystander for a dry piece of clothing. 4. Determine if the membranes have ruptured.

1

The nurse explains to a client in labor that the fetal heart rate baseline is 150, with accelerations to 165, variable decelerations to 140, and moderate long-term variability. Which statement about the most important part of fetal heart monitoring indicates that the client understands the nurse's teaching? 1. "Presence of variability" 2. "Depth of decelerations" 3. "Fetal heart rate baseline" 4. "Absence of variable decelerations"

1

The nurse is explaining Leopold maneuvers to a client who is in the early stage of labor. What should the nurse explain as being the purpose of the second maneuver? 1. Locate the fetal back 2. Identify the descent of the presenting part into the pelvis 3. Determine if the pelvic inlet contains the head or buttocks 4. Determine if the fetal head or buttocks occupies the uterine fundus

1

The nurse is observing a student provide care to a client who is in early labor. Which student actions should be corrected? 1. Applying a fetal heart monitor followed by an explanation of the reason for its use 2. Upon entering the room, speaking with the client prior to looking at the fetal heart monitor 3. Using layman's terms to provide the client with an explanation of the reason for electronic fetal monitoring 4. Incorporating cues that arise from intuition or from observations of the client and family as opposed to focusing on the fetal heart monitor

1

The nurse is orienting a new graduate nurse to the labor and birth unit. Which statement indicates that teaching about a client admission has been effective?" 1. "A vaginal examination is performed if delivery appears to be imminent." 2. "Her prenatal record is reviewed for indications of domestic abuse." 3. "She will be positioned supine to facilitate a normal blood pressure." 4. "A urine specimen is obtained by catheter to check for protein and ketones."

1

The nurse is preparing to monitor the fetal heart rate (FHR) of a pregnant client. What should the nurse explain to a nursing student as being the baseline (BL) of this heart rate? 1. "The baseline FHR excludes periods of marked variability." 2. "Normal baseline FHR ranges from 100 to 180 beats per minute." 3. "The baseline FHR should include periodic or episodic changes in FHR." 4. "The baseline rate is the mean FHR during a 5-minute period rounded to increments of 5 beats per minute."

1

The student nurse is performing Leopold maneuvers on a client in labor. For which action should the staff nurse intervene? 1. After determining where the back is located, the cervix is assessed. 2. The upper portion of the uterus is palpated, and then the middle section. 3. Following voiding, the client's abdomen is palpated from top to bottom. 4. The client is assisted into the supine position, and the position of the fetus is assessed.

1

Which pattern, if seen on an electronic fetal monitoring strip, should the nurse explain to a client in labor as being a change in the baseline fetal heart rate? 1. Tachycardia 2. Acceleration 3. Late deceleration 4. Sinusoidal pattern

1

The labor and delivery nurse is preparing a prenatal class about facilitating the progress of labor. Which pain response should the nurse identify as most likely to impede progress in labor? 1. Muscle tension 2. Increased pulse 3. Increased respirations 4. Elevated blood pressure

1 Explanation: 1. Muscle tension can impede labor progress by increased oxygen and calorie consumption and by creating a mechanical obstruction that the uterine contractions must overcome to achieve labor progress. 2. Increased pulse is a manifestation of pain, but it does not impede labor. 3. Increased respiration is a manifestation of pain, but it does not impede labor. 4. Elevated blood pressure is a manifestation of pain, but it does not impede labor.

A healthy 18-year-old client who is at 40 weeks' gestation experiences vaginal expulsion of stringy mucus followed by blood-tinged secretions unaccompanied by discomfort or any other changes. Based upon these findings, what is most likely to occur within the next 24 to 48 hours? 1. Onset of labor 2. Cesarean section 3. Chorioamnionitis 4. Spontaneous abortion

1 Explanation: 1. Softening and effacement of the cervix is accompanied by expulsion of the mucous plug and a small amount of blood loss from the exposed cervical capillaries. The resulting pink-tinged secretions are called bloody show. Bloody show is considered a sign that labor will begin within 24 to 48 hours. 2. The client is describing pregnancy-related changes associated with imminent onset of labor. Based upon her report, there is no indication that cesarean section will be necessary. 3. Chorioamnionitis is associated with premature rupture of amniotic membranes (PROM). Based upon the client's report, she is demonstrating mucous plug expulsion and bloody show. 4. The client is most likely demonstrating expulsion of the mucous plug and bloody show, which is considered a sign that labor will begin within 24 to 48 hours.

A client in labor with cervical dilation of 9 cm is experiencing contractions every 2 minutes that are 60 to 90 seconds in duration and is complaining of excruciating rectal pressure. How should the nurse interpret this complaint?1. The client's complaint is congruent with her current stage of labor. 2. The client's complaint may indicate the need for delivery via cesarean section. 3. Based upon the client's complaint, she is experiencing the active phase of labor. 4. The client's complaint is consistent with placental separation, which is normal for her current stage of labor.

1 Explanation: 1. The objective findings and client's complaint are consistent with the transitional phase of labor, during which the client may experience increased rectal pressure as cervical dilatation approaches 10 cm (3.9 in.). 2. The objective findings and client's complaint of increased rectal pressure are consistent with the transitional phase of labor, during which the client may experience increased rectal pressure as cervical dilatation approaches 10 cm (3.9 in.). 3. The objective findings and client's complaint are consistent with the transitional phase of labor, during which the client may experience increased rectal pressure as cervical dilatation approaches 10 cm (3.9 in.). 4. Placental separation occurs after the infant is born.

A client is entering the end of the second stage of labor. What should the nurse expect to assess in this client? Select all that apply. 1. Bulging perineum 2. Parting of the labia 3. Crowning of the fetus 4. Increasing bloody show 5. Increasing rectal pressure

1, 2, 3, 4 Explanation: 1. During the end of the second stage of labor the perineum bulges as the fetal head enters the birth canal. 2. During the end of the second stage of labor the labia part as the fetal head enters the birth canal. 3. During the end of the second stage of labor crowning of the fetal head occurs, which indicates that birth is imminent. 4. During the end of the second stage of labor bloody show will increase. 5. Increasing rectal pressure occurs during the transition stage of labor.

The spouse is concerned that the client in labor will be hungry since the last time any food was eaten was several hours ago. What should the nurse explain as effects of the labor process on the client's gastrointestinal system? Select all that apply. 1. Reduced gastric motility 2. Increased gastric volume 3. Increased gastric motility 4. Reduced absorption of food 5. Prolonged gastric emptying time

1, 2, 4, 5 Explanation: 1. During labor gastric motility is reduced. 2. During labor gastric volume remains increased regardless of the time of the last meal. 3. Gastric motility is decreased, not increased. 4. During labor absorption of solid food is reduced. 5. During labor gastric emptying time is prolonged.

A client in the beginning stages of labor asks the nurse if the labor process will hurt the baby. What should the nurse explain to the client about the fetus's response to labor? Select all that apply. 1. The baby's heart rate will change. 2. The baby feels no pain or sensations. 3. The baby is sensitive to light and sounds. 4. The baby's oxygen level drops about 10%. 5. The blood pressure protects the baby during labor.

1, 3, 4, 5 Explanation: 1. Early fetal heart rate decelerations can occur with intracranial pressures of 40 to 55 mmHg, as the head pushes against the cervix. 2. The fetus is experiencing labor as the woman labors. 3. Beginning at about 37 or 38 weeks' gestation, the fetus is able to experience sensations of light, sound, and touch. 4. Blood flow is decreased to the fetus at the peak of each contraction and fetal oxygen saturation drops about 10%. 5. Fetal blood pressure protects the normal fetus during the anoxic periods caused by the contracting uterus during labor.

The nurse suspects that a pregnant client is experiencing true labor. What did the nurse assess to make this clinical determination? Select all that apply. 1. Contractions increase in intensity. 2. Discomfort occurs in the abdomen. 3. Contractions occur at regular intervals. 4. Walking has no effect on the contractions. 5. Time between contractions gradually becomes shorter.

1, 3, 5 Explanation: 1. In true labor contractions will increase in intensity. 2. In false labor the contractions occur in the abdomen. 3. In true labor the contractions occur at regular intervals. 4. In false labor walking has no effect on the contractions. 5. In true labor the time between contractions gradually becomes shorter.

A client's fetal heart rate has a sinusoidal pattern. What should the nurse consider as being the reason for this pattern? Select all that apply. 1. Fetal anemia 2. Chronic fetal bleed 3. Severe fetal hypoxia 4. Maternal hypotension 5. Umbilical cord compression

1,2,3

The nurse categorizes a client's fetal heart rate tracing as a level 2. What criteria were used for this categorization? Select all that apply. 1. Tachycardic baseline. 2. Decelerations lasted longer than 2 minutes. 3. Scalp stimulation did not effect acceleration. 4. Absent variability with recurrent late decelerations. 5. Variable decelerations that slowly return to baseline.

1,2,3,5

The nurse is caring for a 13-year-old client who is in labor. What actions should the nurse take to support this client's needs? Select all that apply. 1. Provide simple and concrete explanations. 2. Stay with the client during the labor process. 3. Provide soothing encouragement during the transition phase. 4. Provide positive reinforcement with a nonjudgmental manner. 5. Remain calm and provide clear directions during the second stage.

1,2,3,5

The nurse is caring for a high-risk client in the second stage of labor. After which actions should the nurse assess the fetal heart rate? Select all that apply. 1. Vaginal examination 2. Urinary catheterization 3. Ingestion of clear liquids 4. Administration of pain medication 5. Change in oxytocin administration

1,2,4,5

During an antenatal examination the nurse becomes concerned that the client is at high risk. What findings did the nurse use to make this clinical determination? Select all that apply. 1. Smokes one half pack per day of cigarettes 2. Employer provides maternal leave of absence 3. Is estranged from family and the baby's father 4. Loss of 3 lb since last examination 1 month ago 5. Treated for a sexually transmitted infection (STI) 2 months ago

1,3,4,5

The nurse is preparing to admit a pregnant client who is Muslim to the birthing center. What should the nurse keep in mind during the labor process? Select all that apply. 1. Have long-sleeved gowns available. 2. Offer warm fluids to sip during the labor process. 3. Ask the spouse for permission before examining the client. 4. Ensure female healthcare providers examine the client. 5. Provide the spouse with water to cleanse the newborn upon birth.

1,3,4,5

An expectant father has been at the bedside of his laboring partner for more than 12 hours. What would be an appropriate nursing intervention? 1. Insist that he leave the room for at least the next hour. 2. Offer to remain with his partner while he takes a break. 3. Suggest that the client's mother might be of more help. 4. Tell him he is not being as effective as he was and that he needs to let someone else take over.

2

The nurse is admitting a client to the labor and delivery unit. Which aspect of the history requires notifying the healthcare provider? 1. Blood pressure 120/88 2. Dark red vaginal bleeding 3. History of domestic abuse 4. Father is a carrier of sickle-cell trait

2

The nurse is preparing to assess a client who has just arrived in the labor and birth unit. Which statement indicates that additional education is needed? 1. "When you check my cervix, you will find out how thinned out it is." 2. "After you assess my pelvis, you will be able to tell when I will deliver." 3. "You are going to do a vaginal examination to see how far dilated my cervix is." 4. "The reason for a pelvic examination is to determine how low in the pelvis my baby is."

2

The nurse is preparing to assess the fetus of a laboring client. Which should the nurse perform first? 1. Place the client into a left lateral position. 2. Perform the Leopold maneuver to determine fetal position. 3. Dry the maternal abdomen before using the Doppler. 4. Count the fetal heart rate for 30 seconds and multiply by 2.

2

Upon delivery of the newborn, which action most promotes parental attachment? 1. Placing the newborn under the radiant warmer 2. Placing the newborn on the maternal abdomen 3. Taking the newborn to the nursery for the initial assessment 4. Allowing the mother a chance to rest immediately after delivery

2

What should be the nurse's priority when caring for an adolescent in labor? 1. Support persons 2. Developmental level 3. Cultural background 4. Plans for keeping the infant

2

During the fourth stage of labor, the client's blood pressure (BP) is 110/60, pulse 90, and the fundus is firm midline and halfway between the symphysis pubis and the umbilicus. What should the nurse do? 1. Massage the fundus. 2. Continue to monitor. 3. Turn the client onto her left side. 4. Place the bed in Trendelenburg position.

2 Explanation: 1. The uterus should be midline and firm; massage is not necessary. 2. The client's assessment data are normal for the fourth stage of labor, so monitoring is the only action necessary. During the fourth stage of labor, the mother experiences a slight drop in BP and a slightly increased pulse. 3. A left lateral position is not necessary with a BP of 110/60 and a pulse of 90. 4. Trendelenburg position is not necessary with a BP of 110/60 and a pulse of 90.

The primiparous client at 39 weeks' gestation calls the clinic and reports increased bladder pressure but easier breathing and irregular, mild contractions. She also states that she just cleaned the entire house. Which statement should the nurse make? 1. "You should not work so much at this point in pregnancy." 2. "Your body may be telling you it is going into labor soon." 3. "If the bladder pressure continues, come in to the clinic tomorrow." 4. "What you are describing is not commonly experienced in the last weeks."

2 Explanation: 1. There is no indication that the client should decrease her work schedule. 2. One of the premonitory signs of labor includes lightening: The baby drops lower into the pelvis, which creates increased pelvic and bladder pressure but less pressure on the diaphragm, which makes breathing easier. 3. Lightening does not indicate pathology, and therefore there is no need to come to the clinic if the symptoms continue. 4. Lightening is a common and expected finding.

Which client requires immediate intervention by the labor and delivery nurse? 1. Primipara in active labor with urine output of 100 mL/hour 2. Primipara that delivered 1 hour ago with white blood cells (WBCs) of 50,000 3. Multipara at 8 cm, systolic blood pressure has increased 35 mm Hg 4. Multipara at 5 cm with a respiratory rate of 22 between contractions

2 Explanation: 1. This is a normal urine output and requires no further intervention. 2. A white count of 25,000 to 30,000 is normal at the end of labor and during the early postpartum period. This WBC count is abnormally high and requires further assessment and provider notification. 3. The systolic blood pressure will change by up to 35 mm Hg during the first stage of labor and can increase further in the second stage of labor. 4. The respiratory rate increases during labor because uterine contractions increase oxygen requirements. This client requires no further intervention.

The nurse is caring for laboring clients. Which women are experiencing problems related to a critical factor of labor? Select all that apply. 1. Multipara at 3 cm, fetus in longitudinal lie 2. Primipara at 7 cm, fetus in military attitude 3. Multipara at 6 cm, fetus at −2 station, mild contractions 4. Primipara at 5 cm, fetal presenting part is right shoulder 5. Primipara at 4 cm, fetus with macrocephaly due to hydrocephalus

2, 3, 4, 5 1. Lie refers to the relationship between the cephalocaudal axis of the mother and the cephalocaudal axis of the fetal body; longitudinal lie is normal. 2. Attitude refers to the relationship of the fetal parts to one another. Military attitude is an unflexed neck; normal fetal attitude is flexion of the neck. Military attitude creates a larger diameter of the head fitting through the pelvis. This client is experiencing a problem between the maternal pelvis and the presenting part. 3. Station refers to how low in the pelvis the baby's presenting part is; −2 station is high in the pelvis. Contractions should be strong to cause fetal descent and cervical dilation. Mild contractions will not move the baby down or open the cervix. This client is experiencing a problem between the maternal pelvis and the presenting part. 4. The presenting part is the fetal part coming through the cervix. The occiput or back of the baby's head is the most common and most effective presenting part. A shoulder presentation cannot deliver vaginally and will require a cesarean birth. This client is experiencing a problem between the maternal pelvis and the presenting part. 5. Hydrocephalus can lead to macrocephaly, or an abnormally large head. Macrocephalic babies might not fit through the bony pelvis and could require birth by cesarean section. This client is experiencing a problem between the maternal pelvis and the presenting part.

A client entering the third trimester of labor is concerned about having enough muscle strength and stamina to give birth. Which exercises should the nurse review with the client to facilitate the birthing process? Select all that apply. 1. Yoga 2. Pelvic tilt 3. Pelvic rock 4. Kegel exercises 5. McRoberts exercises

2, 3, 4, 5 Explanation: 1. Yoga is not identified as an exercise to facilitate the birthing process. 2. Body-conditioning exercises, such as the pelvic tilt, are taught in childbirth preparation classes. 3. Body-conditioning exercises, such as the pelvic rock, are taught in childbirth preparation classes. 4. Body-conditioning exercises, such as the Kegel exercises, are taught in childbirth preparation classes. 5. Exercises aimed at adducting the legs into an extended McRoberts position help enable the woman to stretch her hamstring muscles, a task usually required during the second stage of labor.

The nurse is teaching a prenatal class. What characteristics about false labor should the nurse include? Select all that apply. 1. Increased thin vaginal secretions 2. Pain in the abdomen that does not radiate 3. Progressive cervical effacement and dilatation 4. Contractions that do not intensify while walking 5. An increase in the intensity and frequency of contractions

2, 4 Explanation: 1. True labor results in an increase in vaginal secretions. 2. True labor results in pain beginning low in the abdomen and radiating upward or into the back. 3. True labor results in progressive dilation. 4. True labor contractions intensify while walking. 5. True labor results in increased intensity and frequency of contractions.

Ketones are present in a urine specimen of a client in the beginning phases of labor. What should the nurse consider as the reason for this laboratory finding? Select all that apply. 1. Edema 2. Vomiting 3. Dehydration 4. Preeclampsia 5. Insulin resistance

2,3,5

The nurse is caring for a client in the second stage of labor. What assessment findings indicate that birth is imminent? Select all that apply. 1. Drop in blood pressure 2. Increased bloody show 3. Bulging of the perineum 4. Subjective feeling of faintness 5. Uncontrollable urge to bear down

2,3,5

What is the purpose for the client in labor to utilize different breathing techniques? Select all that apply. 1. Reduces pain 2. A source of relaxation 3. A source of distraction 4. Speeds up the delivery process 5. An increased ability to cope with contractions

2,3,5

While palpating contractions, the nurse determines that a client is in the latent phase of labor. What findings did the nurse use to make this determination? Select all that apply. 1. Contractions rated as being moderate to strong 2. Contractions rated as being mild to moderate 3. Contractions occur every 6 minutes lasting for 40 seconds 4. Contractions occur every 2 minutes lasting for 50 seconds 5. Contraction occurs every 10 minutes lasting for 30 seconds

2,3,5

A client in labor did not attend prenatal classes and is experiencing severe pain. In which breathing technique should the nurse instruct the client to help with relaxation and control? Select all that apply. 1. Kussmaul breathing 2. Abdominal breathing 3. Slow-paced breathing 4. Pant-pant-blow breathing 5. Modified-paced breathing

2,4

The nurse is assisting with a precipitous birth. In which order should the nurse perform the following actions after the birth of the fetal head? 1. Instruct the client to push. 2. Suction the baby's mouth, throat, and nose. 3. Exert upward traction to the fetal head to facilitate birth of the posterior shoulder. 4. Exert downward traction on the fetal head to facilitate movement of the anterior shoulder.

2,4,3,1

A low-risk client is in the second stage of labor. What actions should the nurse take regarding the auscultation of this client's fetal heart rate? Select all that apply. 1. Evaluate after ambulation. 2. Assess heart sounds every 15 minutes. 3. Evaluate before the membrane rupture. 4. Evaluate before providing medications. 5. Count for 30 seconds and multiply times 2.

2,4,5

Five minutes after delivery, the neonate's body is pink with blue extremities. The heart rate is 150. The infant demonstrates a vigorous cry and good respiratory effort, and is actively moving. His elbows and hips are flexed, with his knees positioned up toward his abdomen. When the nurse flicks the soles of his feet, the neonate withdraws his leg. Which nursing interventions are appropriate? 1. Rescue breathing and stimulation 2. Stimulation and resuscitative efforts 3. Nasopharyngeal suctioning and blow-by oxygen 4. Oxygen via face mask and endotracheal suctioning

3

One minute after delivery the following is assessed in a neonate: heart rate 120 beats per minute, vigorous cry, actively moving, resists attempts to straighten an arm, facial grimace with sole flicking, body pink, extremities blue. What Apgar score should the nurse assign to this infant? 1. 6 2. 7 3. 8 4. 9

3

The client in labor has moderately strong contractions lasting 60 seconds every 3 minutes. The fetal head is presenting at a −2 station. The cervix is 6 cm and 100% effaced. The membranes spontaneously ruptured prior to admission, and clear fluid is leaking. Fetal heart tones are in the 140s with accelerations to 150. Which nursing action has the highest priority? 1. Obtain a clean-catch urine specimen. 2. Apply an internal fetal scalp electrode. 3. Keep the client on bed rest at this time. 4. Encourage the husband to remain in the room.

3

The neonatal nurse specialist is describing neonatal care to nursing students. What statement should the specialist include when describing a proper method for preventing heat loss in the neonate? 1. "After delivery, the newborn is immediately placed in skin-to-skin contact with the mother." 2. "Immediately after delivery, the newborn is wrapped in a blanket and placed on the mother's chest." 3. "If the newborn is under a radiant-heated unit, the neonate is dried, placed on a dry blanket, and left uncovered under the radiant heat." 4. "If a radiant-heated unit is used to keep the neonate warm, the neonate is dried, wrapped in a dry blanket, and placed under the radiant heat."

3

A client whose cervix is dilated 8 cm is restless and frequently changing position in an attempt to get comfortable. Which nursing action is most important? 1. Leave the client alone so she can rest. 2. Ask the family to take a coffee and snack break. 3. Reassure the client that she will not be left alone. 4. Encourage the client to have an epidural for pain.

3 Explanation: 1. The client is in the transitional phase of the first stage of labor and will not want to be alone. 2. The client is in the transitional phase of the first stage of labor. The family members might want to take a break, but the client will not want to be alone. 3. Because the client is in the transitional phase of the first stage of labor, she will not want to be left alone; staying with the client and reassuring her that she will not be alone are the highest priorities at this time. 4. The client is in the transitional phase of the first stage of labor. There is no indication that the client wants pain relief.

While caring for a client in labor the nurse determines that the baby's head has internally rotated. The client's spouse asks about other positional changes that will occur during the labor and birth. How should the nurse describe the rest of the cardinal movements for a baby in a vertex presentation? 1. Expulsion, external rotation, and restitution 2. Restitution, flexion, external rotation, and expulsion 3. Extension, restitution, external rotation, and expulsion 4. Flexion, extension, restitution, external rotation, and expulsion

3 Explanation: 1. The fetus changes position in the following order: descent, engagement, flexion, internal rotation, extension, restitution, external rotation, and expulsion. 2. The fetus changes position in the following order: descent, engagement, flexion, internal rotation, extension, restitution, external rotation, and expulsion. 3. The fetus changes position in the following order: descent, engagement, flexion, internal rotation, extension, restitution, external rotation, and expulsion. 4. The fetus changes position in the following order: descent, engagement, flexion, internal rotation, extension, restitution, external rotation, and expulsion.

A pregnant client expresses a desire to use the Lamaze method for the upcoming birth of her child. What should the nurse explain as elements of this birthing process? Select all that apply. 1. Guided imagery 2. Sensory memory 3. Dissociative relaxation 4. Controlled muscle relaxation 5. Differentiated breathing patterns

3, 4, 5 Explanation: 1. Guided imagery is not a technique within the Lamaze method of childbirth. 2. Sensory memory is a technique within the Kitzinger method of childbirth. 3. Dissociative relaxation is used to promote birth as a normal process in the Lamaze method of childbirth. 4. Controlled muscle relaxation is used to promote birth as a normal process in the Lamaze method of childbirth. 5. Specified breathing patterns are used to promote birth as a normal process in the Lamaze method of childbirth.

A client states that her water broke 2 hours ago. What findings should the nurse identify as indications of normal labor? Select all that apply. 1. Protein of +1 in urine 2. Maternal pulse of 160 3. Blood pressure of 120/80 4. Odorless, clear fluid on underwear 5. Fetal heart rate (FHR) of 130 with average variability

3,4,5

A client's fetal heart rate tracing has a consistent late deceleration pattern. What actions should the nurse take at this time? Select all that apply. 1. Prepare for cesarean birth. 2. Increase intravenous fluids. 3. Monitor maternal blood pressure. 4. Position client on the left side. 5. Apply oxygen 7 to 10 L via face mask.

3,4,5

A client's amniotic fluid is meconium stained. What should the nurse do immediately? 1. Change the client's position in bed. 3. Administer oxygen at 2 L per minute. 2. Notify the healthcare provider that birth is imminent. 4. Begin continuous fetal heart rate monitoring.

4

The fetal heart rate baseline is 140 beats per minute. When contractions begin, the fetal heart rate drops suddenly to 120 and rapidly returns to 140 before the end of the contraction. Which nursing intervention is best? 1. Determine the color of the leaking amniotic fluid. 2. Apply oxygen to the client at 2 L per nasal cannula. 3. Notify the operating room of the need for a cesarean birth. 4. Assist the client to change from the Fowler position to the left lateral position.

4

A 25-year-old woman who is at 38 weeks' gestation with her first pregnancy is embarrassed because of going to the hospital 3 times in the last week for false labor. How should the nurse respond? 1. "Do not feel bad. Everyone makes mistakes sometimes." 2. "It is impossible to distinguish between false labor and true labor." 3. "We will discuss the differences between true labor and false labor so this does not happen again." 4. "It is very difficult to tell the difference between true and false labor. Please know we are here to take care of you whenever you need us."

4 Explanation: 1. Instead of reinforcing the woman's perception of having made an error, the nurse should reassure her that her embarrassment is unwarranted. 2. While it may be difficult to subjectively distinguish between false labor and true labor, vaginal examination can be performed to determine if cervical dilatation is occurring. 3. Rather than reinforcing the woman's incorrect interpretation of what she believed to be true labor, the nurse should provide reassurance and ease the woman's embarrassment. 4. Rather than reinforcing the woman's incorrect interpretation of what she believed to be true labor, the nurse should provide reassurance and ease the woman's embarrassment.

The primiparous client at 40 weeks' gestation reports to the nurse that she has had increased pelvic pressure and increased urinary frequency. Which response by the nurse is best? 1. "Come in for an appointment today and we will check everything out." 2. "Unless you have pain with urination, we do not need to worry about it." 3. "This might indicate that the baby is no longer in a head down position." 4. "These symptoms usually mean the baby's head has descended further."

4 Explanation: 1. There is no need for an additional appointment, as increased pelvic pressure and urinary frequency are premonitory signs of labor. 2. Increased pelvic pressure and urinary frequency are premonitory signs of labor. These are not signs of a urinary tract infection. 3. The client is experiencing premonitory signs of labor; the fetus changing to a breech presentation would be experienced as fetal movement that was formerly felt in the upper abdomen but now is down in the pelvis. 4. This is the best response because it most directly addresses what the client has reported. Increased pelvic pressure and urinary frequency are premonitory signs of labor.

The primiparous client has asked the nurse why her cervix has only changed from 1 to 2 cm in 3 hours of contractions occurring every 5 minutes. How should the nurse respond to this client? 1. "What did you expect? You have only had contractions for a few hours. Labor takes time." 2. "When your perineal body thins out, your cervix will begin to dilate much faster than it is now." 3. "The hormones that cause labor to begin are just getting to be at levels that will change your cervix." 4. "Your cervix has also effaced, or thinned out, and that change in the cervix is also labor progress."

4 Explanation: 1. This reply is not therapeutic. Although it is true that this client has only been in early labor for a short time, and it is true that labor for a primipara averages 12 to 24 hours, the nurse must always be therapeutic in all communication. 2. The perineal body thinning primarily occurs during the second stage of labor; it is not expected now. 3. The hormones that cause labor contractions do not directly cause cervical change; the contractions cause the cervix to change. 4. Cervical effacement must be nearly complete before cervical dilation takes place in primips. This is why the labor and birth of a first baby usually take much longer than do subsequent labors and births.

19) The nurse is reviewing the male reproductive system with a group of high school students. Using the diagram select the structure that the nurse should identify as a storage area for spermatozoa.

Answer: Explanation: The epididymis provides a reservoir for maturing spermatozoa. Page Ref: 74

20) The nurse is preparing materials to counsel a couple after a consultation visit at an infertility clinic. On the diagram, select the area on the sperm that the nurse should identify as fertilizing the ovum.

Answer: Explanation: The head's main components are the acrosome and nucleus. The head carries the male's haploid number of chromosomes (23), and it is the part that enters the ovum at fertilization. Page Ref: 75

16) A pregnant client is scheduled for a lower-segment cesarean birth. Place an X on the area of the uterine structure where the nurse should instruct that this incision for the procedure will be placed.

Answer: Explanation: The isthmus is that portion of the uterus between the internal cervical os and the endometrial cavity. The isthmus is about 6 mm above the uterine opening of the cervix and takes on importance in pregnancy because it becomes the lower uterine segment. At birth, this thin lower segment, situated behind the bladder, is the site for lower-segment cesarean births.

18) The nurse is reviewing the anatomy and physiology of the reproductive organs during a prenatal class. On the diagram, select the area that the nurse should identify as being the round ligament.

Answer: Explanation: The round ligaments arise from the sides of the uterus near the fallopian tube insertions. They extend outward between the folds of the broad ligament, passing through the inguinal ring and canals and eventually fusing with the connective tissue of the labia majora. The round ligaments are made up of longitudinal muscle and undergo hypertrophy and increase in both length and diameter during pregnancy. Page Ref: 63

17) After delivery, the nurse notes that a client is having considerable vaginal bleeding. On the diagram provided, identify the area of the myometrium that is responsible for stopping bleeding after birth.

Answer: Explanation: The thick middle layer of the myometrium is made up of interlacing muscle fibers in figure-eight patterns that surround large blood vessels. Their contraction produces a tourniquet-like action on blood vessels to stop bleeding after birth.

18) The nurse is discussing an embryonic structure that forms red blood cells until the liver takes over the process in the developing fetus. Place an X on the diagram to identify the structure that the nurse is discussing.

Answer: Explanation: The yolk sac is small and functions early in embryonic life. It forms primitive red blood cells during the first 6 weeks of development, until the embryo's liver takes over the process. Page Ref: 141

20) The newborn of a client who received butorphanol tartrate (Stadol) 20 minutes before delivery is demonstrating respiratory depression. The infant, weighing 9.9 lb, is prescribed naloxone (Narcan) 0.1 mg/kg. How many milliliters of the medication should the nurse administer to the newborn? (Calculate to the hundredth decimal point.)

Answer: 0.45 mL Explanation: First calculate the newborn's weight in kilograms by dividing the weight in pounds by 2.2, or 9.9/2.2 = 4.5 kg. Then multiply the prescribed dose of 0.1 mg × 4.5 kg = 0.45 mL. The nurse should provide the newborn with 0.45 mL of naloxone. Page Ref: 402

14) How should the nurse interpret a pregnant client's lecithin/sphingomyelin (L/S) ratio finding of 2:1 on amniotic fluid? 1. Fetal lungs are mature. 2. Fetal lungs are still immature. 3. The fetus has a congenital anomaly. 4. The fetus is small for gestational age.

Answer: 1 Explanation: 1. A 2:1 L/S ratio indicates that the risk of respiratory distress syndrome (RDS) is very low and that the fetus's lungs are mature. 2. Early in pregnancy, the sphingomyelin concentration in amniotic fluid is greater than the concentration of lecithin, and so the L/S ratio is low if the fetus's lungs are immature, which is not the case in this instance. 3. The L/S ratio is not a measurement for congenital anomalies. 4. The L/S ratio is not a measurement for size of the fetus. Page Ref: 262

4) A pregnant client's fundal height is 26 cm at 32 weeks' gestation. Why would the healthcare provider schedule this client for sonograms every 2 weeks? 1. Evaluate fetal growth 2. Determine fetal presentation 3. Assess for congenital anomalies 4. Rule out a suspected hydatidiform mole

Answer: 1 Explanation: 1. A person who is at 32 weeks' gestation should measure 32 cm of fundal height. When a discrepancy between fundal height and measurement exists, the purpose of serial ultrasounds is to monitor fetal growth. 2. Fetal presentation would require only one ultrasound. 3. Assessment of anomalies would require only one ultrasound. 4. Ruling out a hydatidiform mole would require only one ultrasound. Page Ref: 253

16) The nurse is leading a session on nutrition for postpartum clients. Which statement indicates that teaching has been effective? 1. "Breastfeeding requires that I eat lots of protein daily." 2. "Because I am breastfeeding, I need a low calcium intake." 3. "Since I am bottlefeeding, I don't have to eat vegetables." 4. "Bottlefeeding moms like me require a high sodium intake."

Answer: 1 Explanation: 1. Breastfeeding clients should consume 65 g of protein daily during the first 6 months of breastfeeding and 62 g daily during the second 6 months. Protein is a major ingredient in breast milk. 2. Breastfeeding requires an increase of 1000 mg per day of calcium—the same amount of calcium that is recommended during pregnancy. 3. Although vitamin intake is not directly related to bottlefeeding, good nutritional habits are important to form while bottlefeeding, because in the future, the baby will be eating what the mother eats. 4. Sodium intake is not increased during bottlefeeding.

2) The nurse who is counseling a group of middle school girls on pregnancy avoidance should include which statement? 1. "Although condom use is growing, there is still an increasing rate of STIs among teens." 2. "It has become far less acceptable to give birth during your teenage years than it used to be." 3. "You have learned enough from your friends and families to understand how pregnancy occurs." 4. "Although sexuality is common in the media, your peer pressure to have sex is not an important factor."

Answer: 1 Explanation: 1. Condom use is increasing, but the rate of sexually transmitted infections (STIs), including HIV, is also rising. Research indicates that young people 15 to 24 years of age make up 25% of the sexually experienced population in the United States. However, they account for nearly half of the new cases of STIs. 2. Society has become more accepting of teen pregnancy, and there are fewer stigmas attached to being a young mother. 3. Formal education on the physiology of the body and conception will decrease the myths and misunderstandings that abound among teens and undereducated adults. 4. Images of sexuality are common in American society: in music lyrics and videos, in advertising, in television shows and movies. Peer pressure to have sex is also common, and is a strong influence on when a teen becomes sexually active. Page Ref: 236

3) The nurse instructs on the importance of niacin during a preconception counseling class. Which food item selected by a participant indicates that teaching about the sources of niacin has been effective? 1. Fish 2. Milk 3. Apples 4. Broccoli

Answer: 1 Explanation: 1. Dietary sources of niacin include meats, fish, and enriched grains. 2. Milk will provide sources of other vitamins; however, it does not contain significant niacin. 3. Apples will provide sources of other vitamins; however, they do not contain significant niacin. 4. Broccoli will provide sources of other vitamins; however, it does not contain significant niacin.

4) It has been identified that a pregnant client's diet is low in zinc. Which food should the nurse suggest to increase intake of this mineral? 1. Yogurt 2. Bananas 3. Cabbage 4. Shellfish

Answer: 4 Explanation: 1. Yogurt is high in other nutrients but does not have significant levels of zinc. 2. Bananas are high in other nutrients but do not have significant levels of zinc. 3. Cabbage is high in other nutrients but does not have significant levels of zinc. 4. Zinc is found in greatest concentration in meats and meat by-products. Enriched grains also tend to be high in zinc.

The nurse is reviewing the assessment findings of a client who is at 35 weeks' gestation. Which data suggest the need for further investigation? 1. Glycosuria 2. Funic souffle 3. Pseudoanemia 4. Melasma gravidarum

Answer: 1 Explanation: 1. Glycosuria (glucose in the urine) during pregnancy may be normal or may indicate gestational diabetes, so it always warrants further testing. 2 Funic souffle is a normal assessment finding associated with the pulsing of blood through the umbilical cord. 3. Physiologic anemia of pregnancy or pseudoanemia is common during pregnancy and is an expected finding. 4. Facial chloasma or melasma gravidarum (also known as the "mask of pregnancy") is a harmless darkening of the skin over the cheeks, nose, and forehead that sometimes accompanies pregnancy.

15) Which phone call should the prenatal clinic nurse return first? 1. Primipara at 32 weeks, reports headache and blurred vision 2. Primipara at 16 weeks, reports increased urinary frequency 3. Multipara at 18 weeks, reports no fetal movement this pregnancy 4. Multipara at 40 weeks, reports sudden gush of fluid and contractions

Answer: 1 Explanation: 1. Headache and blurred vision are signs of preeclampsia, which is potentially life threatening for both mother and fetus. This client has top priority. 2. Increased urinary frequency is common during pregnancy as the increased size of the uterus puts pressure on the urinary bladder. Urinary frequency is expected. If the client were reporting dysuria or hematuria, a urinary tract infection (UTI) would be suspected, but this client is only reporting increased urinary frequency. This client is a lower priority. 3. Fetal movement should be felt by 19 to 20 weeks. Multiparas sometimes feel fetal movement prior to 19 weeks, but the lack of fetal movement prior to 20 weeks is considered normal. This client is a lower priority. 4. A term client who is experiencing contractions and a sudden gush of fluid is in labor. Although laboring clients should be in contact with their healthcare provider for advice on when to go to the hospital, labor at term is an expected finding. This client is a lower priority. Page Ref: 188

The nurse is presenting an in-service to nursing staff regarding the provision of culturally competent client care. Which statement should the nurse include in the presentation? 1. "Developed countries are becoming increasingly more ethnically diverse." 2. "The rituals and customs of a group reflect the values of the dominant culture." 3. "Identification of cultural values is a task that is unrelated to providing culturally sensitive care." 4. "Many immigrants to a new country will adopt the beliefs and practices of the dominant culture."

Answer: 1 Explanation: 1. In many developed countries such as, for example, the United States, Canada, England, and Germany, populations are becoming more and more ethnically diverse as the number of immigrants continues to grow. 2. The rituals and customs of a group are a reflection of the group's values. 3. The identification of cultural values is useful in planning and providing culturally sensitive care. 4. It is not realistic or appropriate to assume that people of another culture will automatically abandon their ways and adopt the practices of the dominant culture.

6) The nurse reviews nutritional requirements with a breastfeeding mother who is concerned that her milk production has decreased. Which statement indicates that further teaching is required? 1. "I have started cutting back on my protein intake." 2. "At least 3 times a day, I drink a glass of milk." 3. "I am drinking a minimum of 8 to 10 glasses of liquid a day." 4. "I try to take a nap in the morning and afternoon when the baby is sleeping."

Answer: 1 Explanation: 1. It is especially important for the breastfeeding mother to consume sufficient calories because inadequate caloric intake can reduce milk volume. The decreased intake of protein represents a decrease in calories, which will decrease milk production. 2. The breastfeeding mother must increase her protein and calcium intake. 3. The breastfeeding mother must consume a minimum of 8 to 10 glasses of liquid per day. 4. It has also been found that adequate rest is necessary for the body to maintain its production of milk.

The partner of a pregnant client at 16 weeks' gestation accompanies her to the clinic. The partner tells you that the baby just does not seem real to him, and he is having a hard time relating to his partner's fatigue and food aversions. Which statement would be best for the nurse to make? 1. "Many men feel this way. Feeling the baby move will help make it real." 2. "My husband had no problem with this. What was your childhood like?" 3. "You might need professional psychologic counseling. Ask your physician." 4. "If you would concentrate harder, you would be aware of the reality of this pregnancy."

Answer: 1 Explanation: 1. Kicking and ultrasound visualization are concrete evidence of the baby's existence and often are turning points in acceptance for partners. 2. The ambivalence and disbelief occur across all socioeconomic groups, in both partners who were fathered well and those who grew up without a father. 3. This reaction is not indicative of psychologic pathology. 4. Ambivalence is common among partners, especially prior to either seeing the baby on ultrasound or feeling the baby kick and move.

12) Which approach to planning educational activities is best suited to a group of pregnant adolescents? 1. Primarily using visual-based presentations during teaching 2. Combining teaching for adolescent mothers of all ages as one group 3. Respecting confidentiality and building trust by avoiding the topics of drug and alcohol abuse 4. Avoiding the inclusion of handouts with bulleted items and white space as part of the teaching

Answer: 1 Explanation: 1. Many teens prefer teaching aids that are visual and that they can handle, such as realistic fetal models. 2. Teaching adolescents in groups according to their ages may be more effective for learning because younger adolescent mothers' parenting skills and emotional needs may differ from those of older adolescent mothers. 3. The nurse should review the risks associated with the use of tobacco, caffeine, drugs, and alcohol, as well as discussing the fetal effects of these substances. 4. Some pregnant teens have low reading levels and tend to prefer handouts and posters that have visual interest, short sentences, bulleted items, and white space. Page Ref: 240

4) A young woman claims that contraception is not needed because she avoids intercourse on the day of ovulation. How should the nurse instruct this client? 1. Sperm survive 48 to 72 hours in the female reproductive tract. 2. After ovulation, ova are considered fertile for about 72 to 96 hours. 3. Sperm are believed to be healthy and highly fertile for at least 5 days. 4. Refraining from intercourse on the day of ovulation will effectively prevent pregnancy.

Answer: 1 Explanation: 1. Sperm survive 48 to 72 hours in the female reproductive tract. 2. Ova are considered fertile for about 12 to 24 hours after ovulation. 3. Sperm are believed to be most fertile for the first 24 hours following entry into the female reproductive tract. 4. Because sperm survive 48 to 72 hours in the female reproductive tract, avoidance of intercourse for 24 hours will not reliably prevent pregnancy from occurring.

14) A client at 18 weeks' gestation thinks she might have been exposed to a toxin at work that could affect fetal development and asks what organs might be affected at this point in the pregnancy. How should the nurse respond to this client? 1. "The brain is developing now and could be affected." 2. "Because you are in the second trimester, there is no danger." 3. "It's best to not worry about possible problems with your baby." 4. "The internal organs like the heart and lungs could be impacted."

Answer: 1 Explanation: 1. Maximum brain growth and myelination are occurring at this point in fetal development. 2. Although the greatest danger from teratogens is during the embryonic stage (the first 8 weeks of pregnancy), the fetus at 20 weeks is still vulnerable to exposure to teratogens. 3. Avoid telling clients not to worry. Doing so indicates to the client that you do not care about their concerns and will end effective communication. 4. The heart, lungs, and other internal organs form during the embryonic state, or the first 8 weeks of pregnancy. During their formation is when they are most likely to be affected by a teratogen.

10) The nurse is creating a teaching poster for pregnant mothers. Which description of fetal development should the nurse include? 1. Most organs are formed by 8 weeks after fertilization. 2. The embryonic stage is from fertilization until 5 months. 3. Four layers of cells form after the embryo is at the ball stage. 4. After fertilization, the cells only become larger for several weeks.

Answer: 1 Explanation: 1. Most organs are formed during the embryonic stage, which lasts from the 15th day after fertilization until the end of the 8th week after fertilization. This is also a critical period because major organs are being developed and teratogens introduced during this time can increase the risk of congenital abnormalities. 2. The embryonic stage begins on the 15th day after fertilization and ends at the completion of the 8th week after fertilization. 3. Three primary germ layers form from the ball of undifferentiated cells, the blastocyst: ectoderm, mesoderm, and endoderm. 4. After fertilization, the cells reproduce by mitosis, resulting in more cells, not larger cells.

8) The clinic nurse is assisting with an initial prenatal assessment. The following findings are present: spider nevi present on lower legs; dark pink, edematous nasal mucosa; mild enlargement of the thyroid gland; mottled skin and pallor on palms and nail beds; heart rate 88 with murmur present. What is the best action for the nurse to take based on these findings? 1. Have the healthcare provider see the client today. 2. Instruct the client to avoid direct sunlight. 3. Document the findings on the prenatal chart. 4. Analyze previous thyroid hormone laboratory results.

Answer: 1 Explanation: 1. Mottling of the skin is indicative of poor oxygenation and a circulation problem. Skin and nail bed pallor can indicate either hypoxia or anemia. These abnormalities must be reported to the healthcare provider immediately. 2. Spider nevi are common in pregnancy due to the increased vascular volume and high estrogen levels. Nasal passages can be inflamed during pregnancy from edema, caused by increased estrogen levels. 3. These abnormalities must be reported to the healthcare provider immediately. 4. The thyroid gland increases in size during pregnancy due to hyperplasia. Page Ref: 178

13) The newborn of a client who received nalbuphine hydrochloride (Nubain) for pain control was born less than an hour after the medication was given and is exhibiting signs of respiratory depression. Which medication should the nurse prepare to administer to the newborn? 1. Naloxone (Narcan) 2. Fentanyl (Sublimaze) 3. Pentobarbital (Nembutal) 4. Butorphanol tartrate (Stadol)

Answer: 1 Explanation: 1. Narcan is an opiate antagonist, which would reverse the effects of the Nubain. 2. Fentanyl is a short-acting opiate that has been used during labor to relieve pain and induce sedation. It is not an opiate antagonist. 3. Pentobarbital is a sedative. It is not an opiate antagonist. 4. Butorphanol tartrate is a synthetic agonist-antagonist opioid analgesic agent. It is not an opiate antagonist.

7) A client experiencing contractions every 8 to 20 minutes that last 20 to 30 seconds requests pain medication. What should the nurse state as the effect of analgesics given at this time? 1. Prolonged labor 2. Maternal hypotension 3. Fetal respiratory depression 4. Decreased analgesic effectiveness at the end of labor

Answer: 1 Explanation: 1. Pain medication given before labor becomes established is likely to prolong the labor process. 2. Analgesics might lower the blood pressure, but this effect does not cause the contraction pattern to be altered. 3. Pain medication given before established labor does not cause fetal respiratory depression unless the client delivers within an hour of receiving the medication. This is not likely if labor is not established. 4. Medication given early in the labor process does not become less effective at the end of labor.

4) While developing a conference for adolescents, the nurse prepares a handout describing socioeconomic and cultural factors that contribute to adolescent pregnancy. Which information should the nurse include in the handout? 1. Poverty is a major risk factor for teen pregnancy. 2. All cultures share an aversion to early pregnancy. 3. A child born to a teenage mother is at a lower risk for teen pregnancy. 4. The younger the teen when she first gets pregnant, the less likely she is to have another pregnancy in her teens. Answer: 1

Answer: 1 Explanation: 1. Poverty is a major risk factor for teen pregnancy. Adolescents who do not have access to middle-class opportunities tend to maintain their pregnancies because they see pregnancy as their only option for adult status. Teens who are on a low economic trajectory are more likely to become pregnant because of the lack of economic opportunity and the social marginalization that comes with poverty. 2. Early pregnancy is desirable in some cultures, such as where Islam is the predominant religion, where large families are desired, where social change is slow in coming, and where most childbearing occurs within marriage. 3. Daughters and sisters of a woman who had a baby in her early teens tend to have intercourse earlier and are at higher risk for teen pregnancy themselves. 4. The younger the teen when she first gets pregnant, the more likely she is to have another pregnancy in her teens. Page Ref: 235

15) A woman is experiencing mittelschmerz and increased vaginal discharge. Her temperature has increased by 0.6°C (1.0°F) for the past 36 hours. What should these findings indicate to the nurse? 1. Ovulation has occurred. 2. Ovulation will occur soon. 3. Menstruation is about to begin. 4. She is pregnant and will not menstruate.

Answer: 1 Explanation: 1. Signs that ovulation has occurred include: pain associated with rupture of the ovum (mittelschmerz), increased vaginal discharge, and a temperature increase of 0.6°C (1.0°F) over the past 36 hours. 2. A temperature increase does not occur before ovulation has occurred. 3. A temperature increase does not occur when menstruation is about to begin. 4. Pregnancy can be detected only through testing the urine or serum for the presence of human chorionic gonadotropin hormone.

The mother of a client who is 14 weeks pregnant is uncertain about how to be a good grandmother to this baby due to working full time and being so busy. How should the nurse respond in this situation? 1. "How do you envision your role as grandmother?" 2. "Don't worry. You'll be a wonderful grandmother. It will all work out fine." 3. "As long as there is another grandmother available, you do not have to worry." 4. "Grandmothers are supposed to be available. You should retire from your job."

Answer: 1 Explanation: 1. Supportive, nonjudgmental exploration of the client's concerns is one component of therapeutic communication and is appropriate. 2. Casual and/or false reassurance is not appropriate. Effective therapeutic communication requires supportive, nonjudgmental exploration of the client's concerns. 3. Minimizing a client's concern is not appropriate. Effective therapeutic communication requires supportive, nonjudgmental exploration of the client's concerns. 4. Assignment of guilt is not appropriate. Effective therapeutic communication requires supportive, nonjudgmental exploration of the client's concerns

15) The nurse is assisting an expectant couple in developing a birth plan. Which instructions should the nurse include when teaching about this plan? 1. It is a communication tool between the client and the healthcare provider. 2. It is a legally binding contract between the client and the healthcare provider. 3. It allows the client to make choices about the birth process; however, these choices cannot be altered. 4. It includes only client choices and does not take into account standard choices of the healthcare provider.

Answer: 1 Explanation: 1. The birth plan is used as a tool for communication among the expectant parents, the healthcare provider, and the healthcare professionals at the birth setting. 2. It is not a legal document. 3. The written plan identifies options that are available; thus, it can be altered. 4. The birth plan is used as a tool for communication among the expectant parents, the healthcare provider, and the healthcare professionals at the birth setting. Page Ref: 197

4) A pregnant client asks, "What's the difference between the true pelvis and the false pelvis?" How should the nurse respond? 1. "The false pelvis helps support the weight of the pregnant uterus." 2. "The false pelvis consists of the inlet, the pelvic cavity, and the outlet." 3. "The true pelvis does not affect fetal passage during labor and childbirth." 4. "The true pelvis helps direct the presenting fetal part into the false pelvis."

Answer: 1 Explanation: 1. The false pelvis helps support the weight of the pregnant uterus. 2. The true pelvis consists of the inlet, the pelvic cavity, and the outlet. 3. The size and shape of the true pelvis must be adequate for normal fetal passage during labor and childbirth. 4. The false pelvis helps direct the presenting fetal part into the true pelvis.

10) The nurse is explaining the menstrual cycle to a group of women. In which phase should the nurse instruct that the corpus luteum begins to degenerate, estrogen and progesterone levels fall, and the blood supply to the endometrium is reduced? 1. Ischemic phase 2. Secretory phase 3. Menstrual phase 4. Proliferative phase

Answer: 1 Explanation: 1. The ischemic phase is characterized by ischemia of the endometrium. 2. The secretory phase involves glycogen secretion by the endometrium after ovulation. 3. The menstrual phase is the menses. 4. The proliferative phase is characterized by proliferation of the endometrium.

During an interview the nurse asks the partner of a woman in the second trimester of pregnancy what changes they have noticed during the pregnancy. Which answer would indicate a typical response to pregnancy? 1. "She daydreams about what kind of parent she is going to be." 2. "I have not noticed anything. I just found out she is pregnant." 3. "She has been having dreams at night about misplacing the baby." 4. "She has been more tense and anxious than usual, and she is not sleeping well."

Answer: 1 Explanation: 1. The second trimester usually brings increased introspection and consideration of how she will parent. 2. In the first trimester, pregnant women usually tell their partners of the pregnancy, explore their relationship with their mother, and think about their own role as a mother. 3. The needs of the newborn typically are not considered until the third trimester, at which time dreams of misplacing the baby or being unable to get to the baby also may be common. 4. During the third trimester, the woman typically experiences more anxiety and tension, as well as increased discomfort and insomnia

12) The nurse is preparing a handout on the ovarian cycle for a group of middle school girls. Which information should the nurse include? 1. There are two phases of the ovarian cycle: luteal and follicular. 2. The hormone human chorionic gonadotropin (hCG) stimulates ovulation. 3. The ovum travels from the ovary to the tube during the luteal phase. 4. Irregular menstrual cycles have varying lengths of the follicular phase.

Answer: 1 Explanation: 1. The two phases of the ovarian cycle are follicular (days 1 to 14 of the menstrual cycle) and luteal (days 15 to 28 of the menstrual cycle). 2. hCG is secreted by a fertilized ovum and does not stimulate ovulation. 3. The ovum is released from the graafian follicle of the ovary and travels to the fallopian tube during the follicular phase of the ovarian cycle. 4. Menstrual cycles that are irregular in length have a consistent follicular phase but a varying luteal phase.

A client at 30 weeks' gestation is tearful at the time of her follow-up visit. She tells the prenatal clinic nurse that she is excited to finally become a mother and that she has been thinking about what kind of parent she will be. However, she is upset because her mother has told her that she does not want to be a grandmother because she does not feel old enough. Meanwhile, the client's husband has said that the pregnancy does not feel real to him yet and that he will become excited when the baby is actually here. What is the most likely explanation for what is happening within this family? 1. Family members are adjusting to the role change at their own pace. 2. Her mother is rejecting the role of grandparent and will not help out. 3. Her husband will not attach with this child and will not be a good father. 4. The client is not progressing through the developmental tasks of pregnancy.

Answer: 1 Explanation: 1. This is a true statement. When the other family members are at different stages of adjustment to the pregnancy, conflict can ensue. 2. Adaptation to the role of grandparent is another life task that takes time. Younger grandparents often have busy and full lives and view grandparenthood as a time for elderly people who are retired and slowed down. The family will form a view of grandparenthood within this family, in a way that works for them. 3. The husband's statement is quite common. Partners often feel that a pregnancy is not real to them because they are not experiencing any of the physical changes associated with pregnancy. 4. This is a false statement. The client is at the stage of seeking acceptance of this child by others, which first will be her partner and other family members.

1) The pregnant client has completed the prenatal questionnaire and asks the nurse why this form had to be filled out. Which response is the most appropriate? 1. "We occasionally identify a health problem that puts the current pregnancy at higher risk." 2. "This form is designed to predict who will develop problems with their pregnancy or delivery." 3. "The doctor wants all of the pregnant clients to fill out the form so that our records are complete." 4. "Some people have things that have happened in the past that could impact their current pregnancy."

Answer: 1 Explanation: 1. This is the reason for risk assessment during pregnancy, whether it is a client-completed questionnaire or a nurse assessment form. 2. The form will identify those clients who have risk factors based on their medical history; prediction implies seeing into the future without a basis for the concern. 3. The purpose of the form is to identify which clients have risk factors; the fact that records are complete is less important than identifying at risk pregnancies. 4. Although this is true, this statement is too vague to be the best response. It is best to explain specifically that the impact on the current pregnancy might put the pregnancy at higher risk.

3) The nurse is preparing a client in her second trimester for a three-dimensional ultrasound examination. Which statement indicates that teaching had been effective? 1. "I might be able to see who the baby looks like with the ultrasound." 2. "If the ultrasound is normal, it means my baby has no abnormalities." 3. "The nuchal translucency measurement will diagnose Down syndrome." 4. "Measuring the length of my cervix will determine if I will deliver early."

Answer: 1 Explanation: 1. Ultrasounds provide a very clear photo-like image of the fetus, often providing parents the opportunity to identify a familial characteristic such as nose shape. 2. Not all fetal anomalies are detectable by ultrasound. 3. Nuchal translucency measurements are screening, not diagnostic, for trisomies 13, 18, and 21. 4. Transvaginal ultrasound is used to measure the cervical length as a screening for risk of preterm labor. However, a normal-length cervix does not preclude preterm birth. Page Ref: 254

11) Which statement from the mother of a pregnant 13-year-old would be an expected response? 1. "We had such high hopes for you." 2. "I told you that boy was up to no good." 3. "But she was always an easygoing child." 4. "This is just one of those things that happen."

Answer: 1 Explanation: 1. When an adolescent pregnancy is first revealed to the teen's mother, the result is often anger, shame, or disappointment. The degree of negative response will be determined by the age of the teen, the family expectations for the teen, and the presence or absence of other teen pregnancies in the family or support network. In early adolescence, the teen's mother frequently accompanies her daughter to prenatal examinations. The role of the nurse is to facilitate communication between mother and daughter and provide education for both. 2. When an adolescent pregnancy is first revealed to the teen's mother, the result is often anger, shame, or disappointment. 3. When an adolescent pregnancy is first revealed to the teen's mother, the result is often anger, shame, or disappointment. 4. When an adolescent pregnancy is first revealed to the teen's mother, the result is often anger, shame, or disappointment. Page Ref: 239

10) The blood pressure of a client receiving continuous epidural anesthesia for labor has changed from 132/78 mmHg to 78/42 mmHg. What action should the nurse perform first? 1. Administer oxygen. 2. Administer ephedrine 5 to 10 mg intravenously. 3. Verify the client is positioned to promote left uterine displacement. 4. Increase the flow rate of infusion of intravenous crystalloid solution

Answer: 1 Explanation: 1. If hypotension occurs secondary to epidural anesthesia, the nurse should apply oxygen via face mask first. 2. The nurse should notify the anesthesiologist for treatment orders before administering ephedrine. 3. Verification of body position is not identified as a step in the treatment of acute hypotension with an epidural infusion. 4. Administering a bolus of crystalloid fluid occurs after oxygen is applied to the client.

21) A 38-year-old pregnant client is reluctant to attend prenatal classes because the other participants will be much younger. What should the nurse do to encourage this client's attendance at classes? Select all that apply. 1. Support the client's strengths. 2. Find a class with participants that are older. 3. Encourage the client to read prenatal training material online. 4. Nothing, since the client most likely has already researched the birthing process. 5. Prepare a list of reasons for the late pregnancy to use as responses when others ask.

Answer: 1, 2 Explanation: 1. Even though women who are over age 35 and having their first baby tend to be better educated than other healthcare consumers, it should not be assumed that anticipatory guidance and support are not needed. Instead, support the client's strengths and be sensitive to her individual needs. 2. Older expectant parents often feel uncomfortable in classes in which most of the participants are much younger. Because of this, classes for expectant parents over age 35 are now available in many communities. 3. The client needs support and not just information that can be read over the Internet. 4. The older client may be better informed. However, she still needs support. 5. There is no reason for the client to have a list of excuses for delaying pregnancy. Page Ref: 245-246

24) The nurse is preparing teaching for a pregnant client with a history of preterm labor. What information should the nurse specifically provide regarding sexual activity? Select all that apply. 1. Avoid intercourse. 2. Avoid nipple stimulation. 3. Avoid intercourse if vaginal bleeding occurs. 4. Avoid intercourse after the membranes rupture. 5. There are no restrictions to intercourse during pregnancy.

Answer: 1, 2 Explanation: 1. Women with a history of preterm labor may be advised to avoid intercourse because the oxytocin that is released with orgasm stimulates uterine contractions and may trigger preterm labor. 2. Because oxytocin is also released with nipple stimulation, fondling the breasts may also be contraindicated in women with a history of preterm labor. 3. All pregnant women should be instructed to avoid intercourse if vaginal bleeding occurs. 4. All pregnant women should be instructed to avoid intercourse after the membranes rupture. 5. Women with a history of preterm labor may be advised to avoid intercourse because the oxytocin that is released with orgasm stimulates uterine contractions and may trigger preterm labor. Page Ref: 214

17) A client in labor received a dose of meperidine (Demerol) for pain control. Which assessment findings should the nurse suspect are adverse effects of this medication? Select all that apply. 1. Nausea 2. Pruritus 3. Sedation 4. Bradycardia 5. Hypotension

Answer: 1, 2, 3 Explanation 1. Nausea is an adverse effect of meperidine. 2. Pruritus is an adverse effect of meperidine. 3. Sedation is an adverse effect of meperidine. 4. Bradycardia is an adverse effect of fentanyl. 5. Hypotension is an adverse effect of nalbuphine hydrochloride and fentanyl

15) The nurse is preparing a presentation about causes of fetal organ malformation in the first trimester. What prenatal influences on the intrauterine environment should be included in this teaching? Select all that apply. 1. The use of drugs 2. Maternal nutrition 3. The use of saunas or hot tubs 4. Age of the mother at conception 5. The quality of the sperm or ovum

Answer: 1, 2, 3 Explanation: 1. Many drugs can have teratogenic effects. 2. Maternal nutrition, if deficient, can cause damage to the fetus. Vitamins and folic acid taken prior to and during the pregnancy can have beneficial effects. 3. The use of saunas or hot tubs is associated with maternal hyperthermia and neural tube defects. 4. A maternal age of 35 or older is associated with genetic defects that occur at conception, not with first-trimester organ malformation. 5. The quality of the sperm or ovum can affect fertility but not organ formation.

26) A 25-year-old client with a learning disability arrives for her first prenatal visit at week 24 of gestation. What should the nurse emphasize when caring for this client? Select all that apply. 1. Ask if the client has any questions. 2. Avoid rushing through the examination. 3. Provide information in small increments. 4. Ask who is going to financially assist the client. 5. Suggest discussing the pregnancy with social services.

Answer: 1, 2, 3 Explanation: 1. The client may have a lower reading and comprehension level. Asking if the client has any questions will help with retention. 2. The client will be anxious, which can be helped by not rushing through the examination. 3. Providing information in small increments will help with comprehension. 4. The client's financial situation is inappropriate for the nurse to assess. 5. Women with intellectual disabilities are at increased risk of intervention by social services, which may result in the removal of the newborns from their custody. Discussing the pregnancy with social services will add to this client's anxiety. Page Ref: 246

19) The nurse is reviewing the structures of the developing fetus with a group of pregnant clients. What should the nurse emphasize about the function of the placenta? Select all that apply. 1. It produces hormones. 2. It produces glycogen, cholesterol, and fatty acids. 3. It maintains fetal respiration, nutrition, and excretion. 4. It provides a circulatory pathway from the chorionic villi to the embryo. 5. It contains a specialized mucoid connective tissue known as Wharton jelly.

Answer: 1, 2, 3 Explanation: 1. The placenta produces human chorionic gonadotropin (hCG); human placental lactogen (hPL), also referred to as human chorionic somatomammotropin (hCS); relaxin; inhibin; and estrogen and progesterone. 2. The placenta produces glycogen, cholesterol, and fatty acids continuously for fetal use and hormone production. 3. The placental functions include fetal respiration, nutrition, and excretion. To carry out these functions, the placenta is involved in metabolic and transfer activities. 4. The umbilical cord provides a circulatory pathway from the chorionic villi to the embryo. 5. The umbilical cord contains a specialized mucoid connective tissue known as Wharton jelly, which prevents compression of the umbilical cord in utero. Page Ref: 145

22) A pregnant client is reviewing the tentative buffet menu for her upcoming baby shower. Which items should the nurse suggest be substituted to ensure the health of the client and developing fetus? Select all that apply. 1. Baked brie with crackers 2. Greek salad with feta cheese 3. Caesar salad with sourdough croutons 4. Salmon pate with toasted bread rounds 5. Grilled orange roughy with summer vegetables

Answer: 1, 2, 3, 4 Explanation: 1. Soft cheese such as brie should be avoided because of the potential for harboring Listeria. 2. Soft cheese such as feta should be avoided because of the potential for harboring Listeria. 3. Salad dressings such as caesar are made with raw eggs, which should be avoided because of the potential for a Salmonella infection. 4. Pates should be avoided because of the potential for harboring Listeria. 5. There are no known bacterial infection issues associated with eating grilled orange roughy and summer vegetables while pregnant.

18) A client in labor needs an emergency cesarean section. What should the nurse include when preparing this client for rapid induction of labor? Select all that apply. 1. Place a wedge under the right hip. 2. Insert an indwelling urinary catheter. 3. Insert an intravenous infusion catheter. 4. Provide a bolus of 1 L of intravenous fluid. 5. Preoxygenate with 3 to 5 minutes of 100% oxygen.

Answer: 1, 2, 3, 5 Explanation: 1. Before induction of anesthesia, a wedge is placed under the woman's right hip to displace the uterus and prevent vena caval compression in the supine position. 2. An indwelling bladder catheter is usually inserted before surgery for women undergoing cesarean birth. 3. Before induction of anesthesia, intravenous fluids are started so that access to the intravascular system is immediately available. 4. A bolus of intravenous fluid is provided prior to an epidural; however, it is not indicated for general anesthesia. 5. Before induction of anesthesia, the client should be preoxygenated with 3 to 5 minutes of 100% oxygen.

22) The nurse is preparing to perform an initial prenatal assessment of a pregnant client who recently immigrated to the United States. Which cultural aspects should the nurse include during the assessment of this client? Select all that apply. 1. Birth rituals 2. Nutritional practices 3. Use of home remedies 4. Expectations to return to work 5. Beliefs about exercise and activity

Answer: 1, 2, 3, 5 Explanation: 1. Birth rituals may vary according to cultural group and should be assessed. 2. Nutritional practices may vary according to cultural group and should be assessed. 3. Use of home remedies may vary according to cultural group and should be assessed. 4. Expectations to return to work are part of anticipatory guidance, from which every pregnant client and family would benefit. 5. Beliefs about exercise and activity may vary according to cultural group and should be assessed. Page Ref: 196-197

19) During a phone call to the clinic the nurse suspects that a pregnant client is experiencing preeclampsia. What manifestations did this client report that caused the nurse to make this clinical determination? Select all that apply. 1. Double vision 2. Epigastric pain 3. Facial swelling 4. Painful urination 5. Severe headache

Answer: 1, 2, 3, 5 Explanation: 1. Double vision is a manifestation of preeclampsia. 2. Epigastric pain is a manifestation of preeclampsia. 3. Facial edema is a manifestation of preeclampsia. 4. Dysuria is a manifestation of a urinary tract infection. 5. Severe headache is a manifestation of preeclampsia. Page Ref: 188

18) A pregnant client's quadruple screen shows a risk for the fetus to have Down syndrome. Which test results within the screen were used to make this determination? Select all that apply. 1. Inhibin-A 2. Alpha-fetoprotein (AFP) 3. Unconjugated estriol (UE) 4. Hemoglobin electrophoresis 5. Human chorionic gonadotropin (hCG)

Answer: 1, 2, 3, 5 Explanation: 1. Higher than normal levels of inhibin-A may indicate that a woman is at increased risk of having a baby with Down syndrome. 2. Lower than normal AFP could indicate that the woman's child is at risk for Down syndrome or trisomy 18. 3. Lower than normal UE may indicate that a woman is at increased risk of having a baby with Down syndrome. 4. Hemoglobin electrophoresis is not a test within the quadruple screen. 5. Higher than normal levels of hCG may indicate that a woman is at increased risk of having a baby with Down syndrome. Page Ref: 188

22) The nurse is preparing a presentation on psychosocial issues of older pregnant clients. What should the nurse include in this presentation? Select all that apply. 1. Social isolation 2. Financial issues 3. Pending mortality 4. Managing adverse effects 5. Family's and friends' attitudes

Answer: 1, 2, 3, 5 Explanation: 1. Older couples facing pregnancy may feel isolated socially. They may feel different because they are often the only couple in their peer group expecting their first baby. 2. The older couple is generally more financially secure, but when their "baby" is ready for college, the older couple may be close to retirement and might not have the means to provide for their child. 3. The older couple may also be forced to face their own mortality. Older expectant parents may confront the issue earlier as they consider what will happen as their child grows. 4. Managing adverse effects would be a part of a presentation on physiologic issues of older pregnant clients. 5. The family's and friends' responses to the pregnancy may be mixed since it will affect relationships and lifestyle. Page Ref: 245

17) The nurse notes that the majority of clients who signed up to attend prenatal classes are over the age of 35. What should the nurse consider as reasons why these clients are this age? Select all that apply. 1. Age of marriage was later. 2. More effective birth control methods. 3. Waited until financially secure before having a family. 4. Needed to care for aging parents before having a family. 5. Wanted to become established in a career before having a family.

Answer: 1, 2, 3, 5 Explanation: 1. Reasons for women choosing to have their first baby after age 35 include marrying later in life. 2. Reasons for women choosing to have their first baby after age 35 include more effective birth control methods. 3. Reasons for women choosing to have their first baby after age 35 include waiting until financially secure before having a family. 4. Caring for aging parents is not identified as a reason for women choosing to have their first baby after age 35. 5. Reasons for women choosing to have their first baby after age 35 include wanting to become established in a career before having a family. Page Ref: 244

18) A 38-year-old client is thrilled to learn of being pregnant with her first child. What should the nurse identify as advantages for the client having a child at this age? Select all that apply. 1. Most likely well-educated. 2. Ready to make a life change. 3. Decision was made deliberately. 4. Child care will be easier at this age. 5. Able to take on the responsibilities of a child.

Answer: 1, 2, 3, 5 Explanation: 1. Single women or couples who delay childbearing until they are older tend to be well educated and financially secure. 2. Some women are ready to make a change in their lives, wanting to stay home with a new baby. 3. Usually, the decision to have a baby at an older age was deliberately and thoughtfully made. 4. Because of their greater life experiences, they also are more aware of the realities of having a child and what it means to have a baby at this age. Child care will not be easier. 5. Many of the women have experienced fulfillment in their careers and feel secure enough to take on the added responsibility of a child. Page Ref: 244

10) The nurse is reviewing clients who would benefit from a biophysical profile (BPP). Which clients should the nurse identify as a priority? Select all that apply. 1. A gravida who is postterm 2. A gravida with intrauterine growth restriction 3. A gravida with mild hypertension of pregnancy 4. A gravida who is experiencing nausea and vomiting 5. A gravida who complains of decreased fetal movement for 2 days

Answer: 1, 2, 3, 5 Explanation: 1. The infant who is postterm might be compromised due to placental insufficiency. 2. The infant who has intrauterine growth problems might be compromised due to placental insufficiency. 3. The BPP is indicated when there is risk of placental insufficiency or fetal compromise because of maternal preeclampsia or eclampsia. 4. Maternal nausea and vomiting is not a criterion for a BPP. 5. The gravida who is experiencing decreased fetal movement for 2 days needs assessment of the placenta and the fetus. Page Ref: 259 BPP -can be performed after 28 weeks -assesses: fetal breathing movement, fetal movements of body or limbs, fetal tone, amniotic fluid volume, reactive non-stress test indications of risk -intrauterine growth restriction -maternal DM -maternal heart disease -maternal chronic HTN -preeclampsia or eclampsia -maternal sickle cell disease -suspected fetal postmaturity -hx of still births -Rh alloimmunixation -abnormal estriol excretion -renal disease -nonreactive NST

19) A 40-year-old client, pregnant with her first child, arrives for a prenatal visit. What perinatal risk factors should the nurse keep in mind when planning this client's care? Select all that apply. 1. Miscarriage 2. Maternal death 3. Perinatal morbidity 4. Potential for multiple births 5. Maternal chronic health conditions

Answer: 1, 2, 3, 5 Explanation: 1. The rate of miscarriage is higher in pregnant women over age 35. 2. The risk of maternal death is higher for women over age 35 and even higher for women age 40 and older. 3. The rate of perinatal morbidity is higher in pregnant women over age 35. 4. There is no evidence to support that pregnant women over the age of 35 or 40 have a greater potential for multiple births. 5. Women over the age of 40 and older are more likely to have chronic medical conditions that can complicate a pregnancy. Page Ref: 244

1) The nurse is explaining the difference between meiosis and mitosis. Which statements should the nurse include? Select all that apply. 1. Mitosis occurs in most of the cells of the body. 2. Meiotic division leads to cells that halve the original genetic material. 3. Meiosis is the division of a cell into two exact copies of the original cell. 4. Meiosis is the process by which gametes, or the sperm and ova, are formed. 5. Mitosis is splitting one cell into two, each with half the chromosomes of the original cell.

Answer: 1, 2, 4 Explanation: 1. Mitosis is how the majority of cells reproduce so that the new cells have the same structure and function as the original. Meiosis only occurs in gametes. 2. Meiosis creates two cells that have half of the chromosomes of the original cell. 3. Meiosis creates two cells that have half of the chromosomes of the original cell. 4. Both sperm and ova are created through meiosis. 5. Mitosis creates two cells that are exact copies of the original cell.

The nurse notes that a client who is 10 weeks pregnant is experiencing changes to the upper respiratory system. What should the nurse explain as being the reason for these changes? Select all that apply. 1. Estrogen-induced edema 2. Hypersecretion of mucus 3. Decreased white blood cell production 4. Vascular congestion of the nasal mucosa 5. Amniotic fluid reducing total fluid volume

Answer: 1, 2, 4 Explanation: 1. Upper respiratory changes in the form of nasal stuffiness and epistaxis may occur because of estrogen-induced edema. 2. Upper respiratory changes in the form of nasal stuffiness and epistaxis may occur because of hypersecretion of mucus. 3. There is no change in white blood cell production. 4. Upper respiratory changes in the form of nasal stuffiness and epistaxis may occur because of vascular congestion of the nasal mucosa. 5. Amniotic fluid is not reducing the client's total fluid volume.

The nurse is reviewing with a pregnant client the skin changes that she might experience during gestation. What should the nurse include in this discussion? Select all that apply. 1. Linea nigra 2. Spider nevi 3. Psoriatic lesions 4. Striae gravidarum 5. Melasma gravidarum

Answer: 1, 2, 4, 5 Explanation: 1. Changes in skin pigmentation occurring during pregnancy are thought to be stimulated by increased estrogen, progesterone, and α-melanocytic-stimulating hormone levels. The skin in the middle of the abdomen may develop a pigmented line, the linea nigra, which usually extends from the pubic area to the umbilicus or higher. 2. Vascular spider nevi, small, bright-red elevations of the skin radiating from a central body, may develop on the chest, neck, face, arms, and legs. They may be caused by increased subcutaneous blood flow in response to elevated estrogen levels. 3. Psoriatic lesions are not typically associated with pregnancy. 4. Striae gravidarum, or stretch marks, may appear on the abdomen, thighs, buttocks, and breasts. They result from reduced connective tissue strength because of elevated adrenal steroid levels. 5. Changes in skin pigmentation occurring during pregnancy are thought to be stimulated by increased estrogen, progesterone, and α-melanocytic-stimulating hormone levels. Melasma gravidarum, also known as the "mask of pregnancy," a darkening of the skin over the forehead and around the eyes, may develop. Melasma is more prominent in dark-haired women and is aggravated by exposure to the sun.

20) During a prenatal visit the nurse notes that a client entering the third trimester has gained a total of 8 lb. What action should the nurse take at this time? Select all that apply. 1. Assess for nausea. 2. Refer to a dietitian. 3. Suggest amniocentesis. 4. Assess nutritional intake. 5. Discuss importance of adequate weight gain.

Answer: 1, 2, 4, 5 Explanation: 1. The nurse should assess for reasons that may restrict the client's intake, such as nausea. 2. The nurse should refer the client to a dietitian for nutritional teaching. 3. An amniocentesis is not indicated at this time. 4. The nurse should discuss the importance of adequate nutritional intake. 5. The nurse should discuss the importance of adequate weight gain to support the developing fetus. Page Ref: 189

25) A client who is at 28 weeks' gestation arrives for her first prenatal examination in a wheelchair. What should the nurse include when assessing this client? Select all that apply. 1. Signs of physical abuse 2. Evidence of mental abuse 3. Teaching about cesarean birth 4. Reason why prenatal care was delayed 5. Language that would hint toward financial abuse

Answer: 1, 2, 4, 5 Explanation: 1. Women with disabilities are at greater risk of being victimized and of sustaining intimate partner violence. Women who rely on their partner for assistance with activities of daily living are at risk for physical abuse. 2. Women with disabilities are at greater risk of being victimized and of sustaining intimate partner violence. Women who rely on their partner for assistance with activities of daily living are at risk for mental abuse. 3. Pregnant women who use a wheelchair will not necessarily need a cesarean birth. 4. Women with disabilities are at greater risk of being victimized and of sustaining intimate partner violence. Women who rely on their partner for assistance with activities of daily living are at risk for having care withheld. 5. Women with disabilities are at greater risk of being victimized and of sustaining intimate partner violence. They are also at risk for financial abuse. Page Ref: 246

19) The nurse is preparing a client in labor for an emergency cesarean section. Which medication should the nurse expect to be prescribed to prevent the effects of aspirated gastric contents? Select all that apply. 1. Famotidine (Pepcid) 2. Cimetidine (Tagamet) 3. Omeprazole (Prilosec) 4. Pantoprazole (Protonix) 5. Metoclopramide (Reglan)

Answer: 1, 2, 5 Explanation: 1. Prophylactic antacid therapy to reduce the acidic content of the stomach before general anesthesia is common practice. Famotidine is used to help empty gastric contents. 2. Prophylactic antacid therapy to reduce the acidic content of the stomach before general anesthesia is common practice. Cimetidine is used to help empty gastric contents. 3. Omeprazole is a proton pump inhibitor and is not identified as being used before general anesthesia to help empty gastric contents. 4. Pantoprazole is a proton pump inhibitor and is not identified as being used before general anesthesia to help empty gastric contents. 5. Prophylactic antacid therapy to reduce the acidic content of the stomach before general anesthesia is common practice. Metoclopramide is used to help empty gastric contents.

23) A pregnant client with rheumatoid arthritis arrives for a prenatal examination. How should the nurse support this client's needs? Select all that apply. 1. Assist with positioning on the examination table. 2. Repeat teaching instructions slowly and succinctly. 3. Assist with changing clothing for the examination. 4. Permit the client privacy while preparing for the examination. 5. Recommend delaying an examination until later in the pregnancy.

Answer: 1, 3 Explanation: 1. A client with arthritis might find examination positions uncomfortable. The nurse should assist this client with positioning. 2. There is no need to repeat teaching instructions slowly and succinctly because the client does not have a learning disability. 3. A client with arthritis might need assistance changing into an examination gown. 4. An adolescent client would want privacy, more so than the client with a physical disability. 5. The pregnant client with a disability should have the same amount and degree of prenatal care. Delaying examinations is not appropriate to suggest. Page Ref: 246

9) The nurse is assessing a 25-year-old primigravida who is 20 weeks pregnant. Which vital signs finding should the nurse report immediately to the physician? 1. Pulse 88/min 2. Respirations 30/min 3. Blood pressure 134/82 4. Temperature 37.4°C (99.3°F)

Answer: 2 Explanation: 1. A slight increase in pulse is an expected finding during pregnancy due to the increased oxygen consumption to support fetal metabolism. 2. Tachypnea is not a normal finding and requires medical care. 3. The blood pressure is within normal limits. 4. A slightly elevated temperature is an expected finding during pregnancy due to the increased oxygen consumption to support fetal metabolism. Page Ref: 177

20) The screening results of a quadruple screening completed on a 37-year-old pregnant client were not within the normal range. For which additional testing should the nurse prepare this client? Select all that apply. 1. Ultrasound 2. Serum glucose 3. Amniocentesis 4. Serum ferritin levels 5. Fetal heart monitoring

Answer: 1, 3 Explanation: 1. If the screening results are not in the normal range, follow-up testing using ultrasound is often indicated. 2. Serum glucose level is not used to detect fetal genetic abnormalities. 3. If the screening results are not in the normal range, follow-up testing using amniocentesis is often indicated. 4. Serum ferritin levels would measure iron level and will not detect fetal genetic abnormalities. 5. Fetal heart monitoring will not detect fetal genetic abnormalities. Page Ref: 245

1) The nurse is preparing an in-service presentation about the role of culture in adolescent pregnancy. What should the nurse include in this presentation? Select all that apply. 1. "Teens with future goals tend to use birth control more consistently." 2. "Most pregnant teens do not have any relatives who had their first child as teens." 3. "Young teens who have a child are more likely to have another while still a teen." 4. "Although the rate has dropped, non-Caucasian teens are more likely to become pregnant." 5. "Eighty-five percent of teen mothers are middle class, and give birth to gain adult status."

Answer: 1, 3, 4 Explanation: 1. Teens with future goals such as college or a job tend to use birth control more consistently compared with other teens; if they become pregnant, they are also more likely to have abortions. 2. Having a mother or a sister who had her first child during adolescence is a risk for a teen to become pregnant. 3. When the first birth occurs in the early teen years, the next birth also is likely to occur prior to adulthood. 4. In the United States, the adolescent birth rate is higher among African American teens and Hispanic teens than among Caucasian teens. 5. When teens in poverty become pregnant, they are more likely to maintain the pregnancy and view the birth as a way to be seen as an adult. Middle-class teens are more likely to have future education and career goals, use contraception, and seek therapeutic abortion if they become pregnant. Page Ref: 235

23) A client in the second trimester of pregnancy is experiencing severe heartburn. What should the nurse explain about the reasons for this health problem? Select all that apply. 1. Decreased gastrointestinal motility 2. Changes in carbohydrate metabolism 3. Increased production of progesterone 4. Relaxation of the esophageal sphincter 5. Displacement of the stomach by the enlarging uterus

Answer: 1, 3, 4, 5 Explanation: 1. Decreased gastrointestinal motility contributes to heartburn. 2. Changes in carbohydrate metabolism contribute to the development of nausea and vomiting. 3. The increased production of progesterone in pregnancy contributes to heartburn. 4. Relaxation of the esophageal sphincter contributes to heartburn. 5. Heartburn during pregnancy appears to be primarily a result of the displacement of the stomach by the enlarging uterus. Page Ref: 205

27) The nurse is preparing prenatal teaching material to support the learning needs of a pregnant client with a learning disability. What should the nurse keep in mind when preparing these materials? Select all that apply. 1. Provide with videotapes. 2. Provide web site addresses. 3. Allow for extra teaching time. 4. Select an easy-to-understand format. 5. Audiotape information from prenatal classes.

Answer: 1, 3, 4, 5 Explanation: 1. Good communication aids such as videotapes should be used. 2. The client has a learning disability and most likely will not be using a computer. 3. Extra teaching time should be provided to ensure all learning needs are met. 4. Easy-to-understand information will facilitate learning. 5. Good communication aids such as audiotapes from prenatal classes should be used. Page Ref: 247

20) The community nurse is providing prenatal care to a client in the home who has been unable to receive traditional prenatal care. Which barriers should the nurse consider when caring for this client? Select all that apply. 1. Location of healthcare facilities 2. View that prenatal care is insignificant 3. Lack of transportation to healthcare facilities 4. Appointment schedule conflicting with work hours 5. Lack of support to care for other children while attending prenatal appointments

Answer: 1, 3, 4, 5 Explanation: 1. Home care is especially effective in removing barriers for women who have difficulty accessing health care. A lack of locally available healthcare facilities is a barrier. 2. Viewing that prenatal care is insignificant is not identified as a barrier to a woman having difficulty accessing health care. 3. Home care is especially effective in removing barriers for women who have difficulty accessing health care. Lack of transportation to healthcare facilities is a barrier. 4. Home care is especially effective in removing barriers for women who have difficulty accessing health care. Appointment schedule that conflicts with work hours is a barrier. 5. Home care is especially effective in removing barriers for women who have difficulty accessing health care. Lack of support to care for other children while attending prenatal appointments is a barrier. Page Ref: 195

19) The nurse is preparing teaching material on chorionic villus sampling (CVS) for a client who is entering the 10th week of gestation. What risks should the nurse include with this material? Select all that apply. 1. Bleeding 2. Embryonic puncture 3. Intrauterine infection 4. Inability to obtain a tissue sample 5. Inadvertent rupture of the membranes

Answer: 1, 3, 4, 5 Explanation: 1. Risks of CVS include bleeding. 2. Embryonic puncture is not a risk associated with CVS. 3. Risks of CVS include intrauterine infection. 4. Risks of CVS include failure to obtain tissue. 5. Risks of CVS include rupture of the membranes. Page Ref: 263

21) The nurse is instructing a newly pregnant client who follows a vegan eating plan about folic acid. What should the nurse include when teaching the client about this nutrient? Select all that apply. 1. The best sources are fresh green leafy vegetables. 2. Overcooking foods high in folic acid is preferred. 3. Foods high in folic acid should be protected from light. 4. Cook foods high in folic acid with small amounts of water. 5. Peanuts and whole-grain breads and cereals are good sources.

Answer: 1, 3, 4, 5 Explanation: 1. Sources for folic acid include fresh green leafy vegetables. 2. To prevent unnecessary loss, foods with folic acid should not be overcooked. 3. To prevent unnecessary loss, foods with folic acid should be stored covered to protect them from light. 4. To prevent unnecessary loss, foods with folic acid should be cooked with only a small amount of water. 5. Sources of folic acid include peanuts and whole-grain breads and cereals, which are all appropriate for the vegan eating plan.

21) The nurse provides teaching on the signs of impending labor with a client at week 37 of gestation. Which client statements indicate that additional teaching would be beneficial? Select all that apply. 1. "A bloody show means labor will begin within 2 days." 2. "Having uterine contractions that increase over time is a sign of labor." 3. "Uterine contractions that do not radiate to the back are a sign of labor." 4. "Having a spontaneous flow of water from my vagina is a sign of labor." 5. "Expulsion of a plug of mucus means the baby has dropped into the pelvis."

Answer: 1, 3, 5 Explanation: 1. Bloody show is a sign of impending labor, which will begin before 2 days. 2. Uterine contractions that increase over time are a sign of impending labor. 3. Uterine contractions that do not radiate to the back describe false labor. 4. Spontaneous rupture of membranes is a sign of impending labor. 5. Expulsion of a mucous plug is a sign of impending labor and not an indication that the fetus has dropped into the pelvis. Page Ref: 192

The manager notes that a neonatal intensive care unit (NICU) nurse is practicing culturally competent care. What did the manager observe to make this decision? Select all that apply. 1. Respects the rituals of the ethnic group of a new mother 2. Explains the processes that are followed in an American hospital 3. Speaks a few phrases in the language of a non-English-speaking client 4. Discusses the odd practices that a client from Europe wants to have done 5. Contacts an interpreter to facilitate communication with a Spanish-speaking client

Answer: 1, 3, 5 Explanation: 1. Evidence of cultural competence includes respecting the rituals of the client's ethnic group. 2. Explaining the processes to be followed in an American hospital can be perceived as being ethnocentric behavior. 3. Evidence of cultural competence includes learning the language, or at least several key phrases, of at least one of the cultural groups with whom the nurse interacts. 4. Discussing a non-American client's odd practices could be perceived as being ethnocentric behavior. 5. Evidence of cultural competence includes providing for the services of an interpreter if a language barrier exists.

21) The nurse is evaluating the effectiveness of prenatal education provided to the spouse of a pregnant client. Which observations indicate that this education was effective? Select all that apply. 1. Client is relaxed. 2. Client and spouse are arguing. 3. Spouse is looking forward to the birth. 4. Spouse expresses fear of being a father. 5. Client is following prenatal recommendations. Answer: 1, 3, 5

Answer: 1, 3, 5 Explanation: 1. Research indicates that increased focus on the father's needs during prenatal care improves the mother's stress levels. 2. Arguing could indicate that the spouse is not transitioning to fatherhood and will affect the client's stress levels. 3. Research indicates that increased focus on the father's needs during prenatal care aids his transition to fatherhood. 4. Fear of being a father indicates that prenatal education was not effective in helping the spouse transition to fatherhood. 5. Research indicates that increased focus on the father's needs during prenatal care improves the mother's prenatal health behavior. Page Ref: 195

20) A client at 20 weeks' gestation is scheduled for a transabdominal ultrasound. What should the nurse instruct this client about the examination? Select all that apply. 1. "The entire procedure takes between 20 and 30 minutes." 2. "Arrive 30 minutes before the examination so pain medication will be effective." 3. "Transmission gel will be spread over the abdomen during the examination." 4. "Use an over-the-counter enema to empty the colon before the examination." 5. "Drink 1.5 quarts of water 2 hours before the exam and refrain from voiding."

Answer: 1, 3, 5 Explanation: 1. Ultrasound testing takes 20 to 30 minutes. 2. Ultrasound is a painless noninvasive diagnostic test. No anesthesia or pain medication is required prior to the procedure. 3. Transmission gel is generously spread over the client's abdomen, and the sonographer slowly moves a transducer over the abdomen to obtain a picture of the uterine contents. 4. It is not necessary to have an empty colon for an ultrasound. 5. The bladder must be full for an ultrasound. The client should be instructed to drink 1 to 1.5 quarts of water 2 hours before the examination and refrain from voiding to ensure a full bladder. Page Ref: 254

5) Which nursing action can prevent or detect common side effects of epidural anesthesia? Select all that apply. 1. Preloading the client with a rapid infusion of IV fluids 2. Continuing the client on oral fluids only to prevent hypotension 3. Assisting the client to empty the bladder before the anesthesia is started 4. Use of intermittent fetal heart rate (FHR) monitoring so the client can use the birthing ball 5. Monitoring the fetal heart rate (FHR) for late deceleration and decrease in rate

Answer: 1, 3, 5 Explanation: 1. Hypotension can be prevented by preloading with rapid IV infusion followed by continuous IV infusion. 2. Hypotension can be prevented by preloading with rapid IV infusion followed by continuous IV infusion. The amount of oral fluids that would be required to prevent hypotension makes this approach inappropriate for the client in labor. 3. The epidural decreases the urge to urinate. The client's bladder should be assessed frequently for distention. 4. Hypotension can be prevented by preloading with rapid IV infusion followed by continuous IV infusion. Variability of FHR and late decelerations can occur if maternal hypotension occurs. Continuing FHR monitoring is essential. 5. Hypotension can be prevented by preloading with rapid IV infusion followed by continuous IV infusion. Variability of FHR and late decelerations can occur if maternal hypotension occurs. Continuing FHR monitoring is essential.

20) The nurse is preparing a teaching guide to be used for prenatal, pregnancy, and postpartum nutritional classes. Which nutrients should the nurse emphasize that remain consistent throughout these gestational periods? Select all that apply. 1. Fat 2. Iron 3. Protein 4. Vitamin K 5. Pyridoxine

Answer: 1, 4 Explanation: 1. Fat requirements are unchanged during pregnancy and should account for about 20% to 35% of daily caloric intake, of which 10% or less should be saturated fat. 2. Iron requirements increase during pregnancy because of the growth of the fetus and placenta and the increased maternal blood volume. 3. The protein requirement for a pregnant woman is 60 g/day, an increase of 14 g over nonpregnant levels. 4. The recommended daily allowance (RDA) for vitamin K, 90 mcg per day, does not increase during pregnancy. 5. The recommended daily allowance (RDA) during pregnancy for pyridoxine is 1.9 mg/day, an increase of 0.6 mg over the allowance for nonpregnant women.

17) The nurse is reviewing amniocentesis with a pregnant client. In which order should the nurse explain the steps that will occur during this procedure? 1. Conduct an ultrasound 2. Fetal heart rate assessed 3. Local anesthetic provided 4. Skin cleansed with antiseptic solution 5. Insertion site observed for fluid streaming 6. 22-gauge needle inserted to withdraw amniotic fluid

Answer: 1, 4, 3, 6, 5, 2 Explanation: 1. An ultrasound is performed first to identify amniotic fluid pockets. 2. Fetal heart rate is assessed last. 3. A local anesthetic is provided after the skin is cleansed. 4. Skin is cleansed after the ultrasound. 5. Fluid streaming occurs after the 22-gauge needle is removed. 6. A 22-gauge needle is inserted after the local anesthetic. Page Ref: 261

24) The nurse is caring for a pregnant client who has scoliosis that has affected sensation below the level of the umbilicus. What should the nurse instruct the client to do, to reduce the risk of adverse effects during pregnancy? Select all that apply. 1. Walk slowly and deliberately. 2. Limit the amount of daily exercise. 3. Eat fewer calories to restrict weight gain. 4. Ingest adequate amounts of fruits, vegetables, and water. 5. Review signs of pending labor other than uterine contractions.

Answer: 1, 4, 5 Explanation: 1. Pregnancy may shift the center of gravity. For the client with lumbar scoliosis, this could cause the client to have difficulty maintaining balance with walking. Walking slowly and deliberately will reduce the risk of falling. 2. There is no reason for the client to restrict the amount of daily exercise. 3. Restricting weight gain while pregnant is not healthy for the client or fetus. 4. Ingesting adequate amounts of fruits, vegetables, and water will prevent the development of constipation, which can occur when nervous innervation to the lower abdomen is affected. 5. The client has reduced sensation below the level of the umbilicus and may not recognize or feel uterine contractions associated with pending labor. The nurse needs to review the other signs of pending labor, such as spontaneous rupture of membranes and bloody show. Page Ref: 247

19) A client who became pregnant at a weight of 60 kg and a BMI of 16 has gained 4 kg at the end of the 5th gestational month. How many more pounds of weight should the nurse counsel this client to gain to achieve the least amount of weight that an underweight person should gain while pregnant?

Answer: 19.2 lb Explanation: For a client with a BMI less than 18.5, the weight gain for the total pregnancy should be between 28 and 40 lb. The client has gained 4 kg, or 8.8 lb. To achieve the minimal amount of weight that the client should gain during the entire pregnancy, subtract the amount of weight gained from the minimal amount, or 28 lb - 8.8 lb = 19.2 lb.

18) A newly pregnant client weighs 75.0 kg and has a body mass index of 28.5. What is the maximum amount of weight in pounds that this client should weigh during the pregnancy?

Answer: 190 lb Explanation: For a client with a BMI between 25.0 and 29.9, the weight gain for the total pregnancy should be between 15 and 25 lb. This client's weight is 75 kg. To determine the maximum amount of weight to gain, first determine the client's weight in pounds by multiplying the weight in kilograms by 2.2, or 75.0 × 2.2 = 165 lb. Then add the maximum amount of weight of 25 lb to this total, or 165 + 25 = 190 lb.

5) The nurse instructs a client about conception and fetal development. Which client statement indicates understanding about transportation time of the zygote through the fallopian tube and into the cavity of the uterus? 1. "It will take 8 days for the egg to reach the uterus." 2. "It will take at least 3 days for the egg to reach the uterus." 3. "It will take 18 hours for the fertilized egg to implant in the uterus." 4. "It will only take 12 hours for the egg to go through the fallopian tube."

Answer: 2 Explanation: 1. "It will take 8 days for the egg to reach the uterus" is an incorrect interpretation of the information on conception. 2. It takes at least 3 days for the egg to reach the uterus. 3. "It will take 18 hours for the fertilized egg to implant in the uterus" is an incorrect interpretation of the information on conception. 4. "It will only take 12 hours for the egg to go through the fallopian tube" is an incorrect interpretation of the information on conception.

10) Which client statement on cultural or religious influences on nutrition requires intervention? 1. "I avoid milk and meat at meals because I am Jewish." 2. "My auntie sent me clay from the south to eat every day." 3. "Because I am Muslim, I do not ever eat any pork products." 4. "My grandmother makes sure I eat a serving of greens each day."

Answer: 2 Explanation: 1. A kosher diet involves avoiding pork and shellfish and not eating dairy and meat at the same meal. 2. Eating clay is pica. The clay, being a type of soil, can be contaminated with hazardous substances and should be avoided. Some African Americans, especially those from the South, practice clay-eating pica. 3. Dietary restrictions in the Muslim tradition include avoidance of pork. Because other meats are eaten, the client is not at risk for protein or iron deficiency. 4. Greens, such as collard greens and spinach, have high amounts of folic acid and are healthy foods to eat during pregnancy. Women from the southern United States often eat greens.

19) The nurse is teaching a couple pregnant for the first time. Which statement made by the couple about prenatal classes indicates that additional information is necessary? 1. "Facilitate better communication between both partners." 2. "Eliminate the risk of needing a vacuum extraction or cesarean birth." 3. "Help parents cope with the discomforts and unknowns of childbirth."

Answer: 2 Explanation: 1. A secondary goal of prenatal classes is to facilitate communication between the pregnant woman and her partner. The primary goals are to inform participants of the birth process and teach them skills to cope with labor and birth. 2. This is false reassurance. There is no guarantee that an operative birth can be avoided, even if a couple attends prenatal classes. 3. This is one of the main goals of prenatal classes: learning skills to help get through the discomforts of childbirth. Participants also learn how the birthing process progresses. Participants will learn new skills that will facilitate the birthing process. 4. One of the strategies used in prenatal classes is to have participants write a birth plan that lists their requests for how they want their birthing experience to be. But putting their desires down in writing is less important than learning about the birth process and learning skills to cope with labor and delivery.

14) A 16-year-old is making her first prenatal visit to the clinic in her fourth month of pregnancy. What is the nurse's first responsibility? 1. Contact the social worker. 2. Develop a trusting relationship. 3. Teach the client about proper nutrition. 4. Schedule the client for prenatal classes.

Answer: 2 Explanation: 1. A social worker might be able to provide assistance with financial program eligibility, support groups, or obtaining baby items such as furniture and car seats. 2. The most important goal for the nurse caring for a pregnant adolescent is to be open minded and nonjudgmental in order to foster trust between the adolescent and the nurse. Through a trusting relationship, the nurse can provide counseling and education to the mother-to-be, both about her body and the fetus. Developing a trusting relationship with the pregnant adolescent is essential. Honesty, respect, and a caring attitude promote self-esteem. 3. Although nutrition is an important physiologic need, without a trusting relationship, little teaching will occur because the teen will often "tune out" adults that she does not trust. 4. Prenatal classes specifically designed for teen mothers and attended by only teen mothers facilitate both learning and support for the teens. Page Ref: 241

5) The nurse is providing nutritional counseling for a postpartum client with a hemoglobin of 8. Which statement indicates that additional teaching is necessary? 1. "I need to increase food sources that contain iron." 2. "If I drink lots of milk, I will increase my iron level faster." 3. "My iron is low, but it will increase as I take iron supplements." 4. "I might feel less energetic and tire more easily while my iron is low."

Answer: 2 Explanation: 1. Anemia requires additional iron. Many foods, such as red meat, will provide iron. Increasing iron-rich foods will improve anemia. 2. Milk does not contain iron; it contains calcium. Increased calcium intake will not increase hemoglobin levels. Further, iron should not be taken with milk, as the iron will not be absorbed. 3. Iron supplements are indicated with anemia. This client's hemoglobin level is 8; lower than 10 is considered anemia during pregnancy. Taking iron will increase hemoglobin. 4. Hemoglobin carries oxygen; when the hemoglobin level is low, the muscles are not adequately oxygenated, especially during activity, and fatigue results.

The nurse has completed a presentation for newly pregnant women about the changes of pregnancy. Which participant's statement reflects accurate comprehension of the information? 1. "Uterine souffle is a positive change of pregnancy." 2. "A positive Goodell sign is a probable change of pregnancy." 3. "Changes in the pelvic organs are presumptive signs of pregnancy." 4. "Three positive pregnancy tests in a 1-week period is considered to be a positive change of pregnancy."

Answer: 2 Explanation: 1. Because uterine souffle can be objectively identified but may be caused by conditions other than pregnancy, it is considered to be a probable change of pregnancy. 2. A positive Goodell sign can be objectively identified but may also be caused by conditions other than pregnancy; therefore, it is considered to be a probable change of pregnancy. 3. Changes in the pelvic organs can be objectively identified; however, because some pelvic organ changes may be associated with conditions other than pregnancy, they are considered to be probable changes of pregnancy. 4. Because other conditions may cause elevated hCG, pregnancy tests are considered probable changes of pregnancy.

18) An expectant couple is determining their compatibility with a healthcare provider. Which question should the nurse encourage the couple to ask first? 1. "Can my children attend the birth?" 2. "What is your philosophy of birth?" 3. "If I have a cesarean birth, can my husband attend?" 4. "What percentage of your clients have episiotomies?"

Answer: 2 Explanation: 1. Children's attendance is a complement to the healthcare provider's philosophy. 2. A thorough understanding of the healthcare provider's philosophy is essential to determining compatibility. 3. A husband's presence at a cesarean birth is a complement to the healthcare provider's philosophy. 4. Episiotomy percentages are a complement to the healthcare provider's philosophy. Page Ref: 198-199

3) The nurse has presented a teaching session on pain relief options to a prenatal class. Which client statement indicates that additional teaching is needed? 1. "An epidural can be continuous or one dose." 2. "General anesthesia is usually recommended for a cesarean section." 3. "Narcotics can be given through a client's epidural infusion catheter." 4. "A pudendal block usually works well to control pain during episiotomy repa

Answer: 2 Explanation: 1. Epidural anesthesia can be administered in a single dose or via continuous infusion. 2. Compared to general anesthesia, spinal anesthesia is usually the anesthetic of choice indicated in the management of clients undergoing cesarean section. 3. To provide analgesia for approximately 24 hours after the birth, the analgesia provider may inject an opioid, such as morphine sulfate (Duramorph) or fentanyl (Sublimaze), into the epidural space immediately after the birth. 4. A pudendal block technique is used in the second stage of labor for the provision of perineal anesthesia for the latter part of the first stage of labor, the second stage, birth, and episiotomy repair.

8) The nurse is presenting a community education session on female hormones. Which participant statement indicates the need for further information? 1. "Estrogen is what causes females to look female." 2. "Prostaglandin is responsible for achieving conception." 3. "Progesterone is present at the end of the menstrual cycle." 4. "The presence of some hormones causes others to be secreted."

Answer: 2 Explanation: 1. Estrogen causes secondary sex characteristics, such as enlarged breasts and widened hips. 2. Prostaglandin is not related to conception. Prostaglandin is called the hormone of pregnancy because it maintains pregnancy. 3. Progesterone is present in large quantities during the secretory phase of the menstrual cycle. 4. An example is that the production of gonadotropin-releasing hormone (GnRH) causes the secretion of luteinizing hormone (LH) and follicle-stimulating hormone (FSH).

9) A woman has been unable to complete a full-term pregnancy because the fertilized ovum failed to implant in the uterus. Which hormone is most likely causing this client's issues with pregnancy? 1. Estrogen 2. Progesterone 3. LH (luteinizing hormone) 4. FSH (follicle-stimulating hormone)

Answer: 2 Explanation: 1. Estrogen primarily assists in maturation of the ovarian follicles and causes endometrial mucosa to proliferate. 2. Progesterone is the likely cause because it decreases uterine motility and contractibility caused by estrogens, thereby preparing the uterus for implantation. 3. LH is a hormone secreted by the pituitary gland. 4. FSH is a hormone secreted by the pituitary gland. Page Ref: 70

13) The nurse is preparing a presentation on the menstrual cycle for a group of high school students. Which statement should the nurse include in this presentation? 1. "One hormone controls the phases of the menstrual cycle." 2. "The secretory phase occurs when a woman is most fertile." 3. "Menstrual cycle phases vary in order from one woman to another." 4. "The menstrual cycle has five distinct phases that occur during the month."

Answer: 2 Explanation: 1. Four hormones control ovulation and therefore the menstrual cycle: progesterone, estrogen, follicle-stimulating hormone (FSH), and luteinizing hormone (LH). 2. During the secretory phase, the endometrium is thickest, and glycogen is produced to nourish a fertilized ovum. 3. Although the length of the menstrual cycle might vary, the phases of the menstrual cycle always occur in the same order. 4. There are four phases of the menstrual cycle: menstrual, proliferative, secretory, and ischemic phases.

3) The nurse is teaching an early pregnancy class for clients in the first trimester of pregnancy. Which statement requires immediate intervention by the nurse? 1. "When my nausea is bad, I will drink some ginger tea." 2. "It is normal for my vaginal discharge to get green colored." 3. "I will urinate less often during the middle of my pregnancy." 4. "The fatigue I am experiencing will improve in the second trimester."

Answer: 2 Explanation: 1. Ginger helps nausea, and is safe for use during pregnancy. 2. Leukorrhea is an increase in white vaginal discharge and is an expected finding during pregnancy. Green discharge is not a normal finding and could indicate an infection. Further assessment is required for a client with green vaginal discharge. 3. As the uterus rises in the pelvis during the second trimester, urinary frequency decreases. Urinary frequency increases again during the end of the third trimester as the fetal head descends into the pelvis. 4. First-trimester fatigue is common; fatigue usually improves during the second trimester. Page Ref: 205

7) The nurse is conducting a postpartum visit to a client who is formula-feeding her infant. Which client statement indicates that teaching about weight maintenance has been effective? 1. "I have increased my caloric intake by 600 calories per day." 2. "My dietician has set my weight loss goal at 1 to 2 pounds per week." 3. "Instead of making another doctor's appointment, I started a diet that my best friend recommended." 4. "My daily regimen includes taking extra vitamin A, vitamin C, and thiamine in order to meet my body's increased need for nutrients after pregnancy."

Answer: 2 Explanation: 1. If the mother has a good understanding of nutritional principles, it is sufficient to advise her to reduce her daily caloric intake by about 300 kcal and to return to prepregnancy levels for other nutrients. 2. Weight loss goals of 1 to 2 pounds (0.45 to 0.9 kg)/week are usually suggested for mothers who formula-feed. 3. The woman should diet only under the guidance of her primary healthcare provider. 4. After birth, the formula-feeding mother's dietary requirements return to prepregnancy levels.

1) The nurse is responding to phone calls. Which call should the nurse return first? 1. 29 weeks' gestation, reports increased fetal movement 2. 37 weeks' gestation, reports no fetal movement for 24 hours 3. 32 weeks' gestation, reports decreased fetal movement for 2 days 4. 35 weeks' gestation, reports decreased fetal movement for 4 hours

Answer: 2 Explanation: 1. Increased fetal movement is not indicative of a problem. 2. A lack of fetal movement in a fetus in the third trimester can indicate fetal hypoxia or fetal death. This client is the highest priority. 3. Although decreased fetal movement can indicate intrauterine growth restriction or fetal hypoxia, this client is not the highest priority. 4. Although decreased fetal movement can indicate intrauterine growth restriction or fetal hypoxia, 4 hours is a very short amount of time to assess decreased fetal movement. Page Ref: 252

A client who is experiencing her first pregnancy has just completed the initial prenatal examination with a certified nurse-midwife. Which statement indicates that the client has a correct understanding of her condition? 1. "The increased size of my uterus means that I am finally pregnant." 2. "Because we heard the baby's heartbeat, I am undoubtedly pregnant." 3. "Since I haven't felt the baby move yet, we don't know if I'm pregnant." 4. "My last period was 2 months ago, which means I'm 2 months along."

Answer: 2 Explanation: 1. Increased uterine size is a probable, or objective, change and does not conclusively verify pregnancy status. 2. Hearing the fetal heart rate is a positive, or diagnostic, change of pregnancy. 3. Fetal movement is a presumptive, or subjective, change of pregnancy. Absence or presence of the sensation of fetal movement is not a conclusive indicator of pregnancy status. 4. Amenorrhea is a presumptive, or subjective, change and does not conclusively verify pregnancy status.

9) The nurse seeks to involve the adolescent father in the prenatal care of his partner. What is the reason for this strategy? 1. Improves the long-term outcome of the relationship 2. Increases the self-care behaviors of the pregnant teen 3. Avoids legal action by the adolescent father's family 4. Avoids conflict between the adolescent father and pregnant teen

Answer: 2 Explanation: 1. Involving the client's partner in prenatal care will not decrease the likelihood that this relationship will be short term. 2. Involving the partner of a pregnant adolescent helps the mother-to-be feel more confident in her decision making and improves her self-confidence and self-esteem, which in turn will improve positive self-care behaviors. 3. The nurse first must explore what the relationship is between the pregnant teen and the father. Relationships between adolescents tend to be short lived, and pregnancy is an added stressor for the couple. 4. If the client desires the participation of her partner, the nurse should provide education and support appropriate to the age, knowledge, and developmental level of the adolescent father. Page Ref: 242

8) A client in the third trimester of pregnancy reports frequent leg cramps. What strategy would be most appropriate for the nurse to suggest? 1. Limit activity for several days. 2. Flex the foot to stretch the calf. 3. Point the toes of the affected leg. 4. Increase intake of protein-rich foods.

Answer: 2 Explanation: 1. Limiting activity is not appropriate. 2. Leg cramps are a common problem in pregnancy, resulting from an imbalance in the calcium-phosphorus ratio; pressure on nerves or decreased circulation in the legs from the enlarged uterus; or fatigue. Dorsiflexing the foot will stretch the calf muscles and will help relieve the cramps. 3. Pointing the toes will exacerbate leg cramps. 4. Protein intake does not affect leg cramps. Page Ref: 209

7) The nurse is caring for a client who is pregnant with twins. Which statement indicates the client needs additional information? 1. "It is rare for twins to both be within the same amniotic sac." 2. "Because both of my twins are boys, I know that they are identical." 3. "If I have one boy and one girl, I will know they came from two eggs." 4. "If my twins came from one fertilized egg that split, they are identical."

Answer: 2 Explanation: 1. Monoamnionic-monochorionic twins are very rare and occur as a result of the zygote splitting 7 or more days after fertilization. 2. Not all same-sex twins are identical or monozygotic, because fraternal, or dizygotic, twins can be the same gender or different genders. 3. The only way to have twins of different genders is if they came from two separate fertilized ova. Monozygotic twins are identical and are the same gender. 4. When the zygote splits, identical twins share the same genotype result.

11) The nurse receives a phone call from a client who thinks she is pregnant. The client reports that she has regular menses that occur every 28 days and last 5 days. The first day of her last menses was April 10. What is the client's estimated date of delivery (EDD)? 1. January 10 2. January 17 3. December 3 4. November 13

Answer: 2 Explanation: 1. Naegele's rule is to add 7 days to the last menstrual period (LMP) and subtract 3 months. The LMP is April 10; therefore, January 17 is the EDD. 2. Naegele's rule is to add 7 days to the last menstrual period (LMP) and subtract 3 months. The LMP is April 10; therefore, January 17 is the EDD. 3. Naegele's rule is to add 7 days to the last menstrual period (LMP) and subtract 3 months. The LMP is April 10; therefore, January 17 is the EDD. 4. Naegele's rule is to add 7 days to the last menstrual period (LMP) and subtract 3 months. The LMP is April 10; therefore, January 17 is the EDD. Page Ref: 184

A 20-year-old client who is at 10 weeks' gestation confides that the pregnancy was unplanned and is unsure about continuing it or sharing news about it with her partner. How should the nurse respond to this client? 1. "You should go to a pregnancy support group to be a good mother." 2. "It's common to feel ambiguous about pregnancy in the first trimester." 3. "These thoughts are because your mother died when you were 4 years old." 4. "It's really unusual for a pregnant woman to feel this way early in the pregnancy."

Answer: 2 Explanation: 1. No psychopathology is present, so a support group is not indicated. 2. Ambivalence toward the pregnancy is very common in the first trimester. 3. Loss of the client's own mother at a young age would not affect the occurrence of ambivalence in the first trimester. 4. Fathers might not be told immediately about the diagnosis of pregnancy.

10) A pregnant client asks what kinds of medications cause birth defects. Which statement would best answer this question? 1. "Almost all medications will cause birth defects in the first trimester." 2. "To be safe, do not take any medication without talking to your doctor." 3. "Too much vitamin C is one of the most common issues but is avoidable." 4. "Birth defects are very rare. Do not worry; your healthcare provider will watch for problems."

Answer: 2 Explanation: 1. Not all medications are teratogenic. 2. Pregnant women need to avoid all medications—prescribed, homeopathic, or over-the-counter—if possible. 3. Vitamin C can cause rebound scurvy but is not teratogenic. 4. The nurse should avoid a "do not worry" answer to ensure therapeutic communication, but it is appropriate to instruct the client to talk to the doctor about medications. Page Ref: 215

A client with a normal prepregnancy weight asks why she has been told to gain 25 to 35 pounds during her pregnancy, but her underweight friend was told to gain 28 to 40 pounds. What should the nurse explain as being the recommended weight gain during pregnancy? 1. More than 25 to 35 pounds for an overweight client 2. More than 25 to 35 pounds for an underweight woman 3. 25 to 35 pounds, regardless of a client's prepregnancy weight 4. The same for a normal-weight woman as for an overweight woman

Answer: 2 Explanation: 1. Overweight women should gain 15 to 25 pounds during pregnancy. 2. Underweight women are encouraged to gain 28 to 40 pounds during pregnancy 3. Prepregnancy weight determines the recommended weight gain during pregnancy. Women of normal weight should gain 25 to 35 pounds during pregnancy for optimal fetal outcome. 4. Overweight women should gain 15 to 25 pounds during pregnancy

15) The nurse has been asked by a community organization to give a presentation on prevention of teen pregnancy. Which statement indicates appropriate steps toward reduction of the local teen pregnancy rate? 1. Classes on how to parent will be mandatory in high school. 2. Plans are made to create a low-cost reproductive health clinic. 3. Parents will be encouraged to avoid discussing sexual activity. 4. Abstinence-only education will be offered in the school and clinics.

Answer: 2 Explanation: 1. Parenting classes for teens who are neither pregnant nor parents do not address reducing teen pregnancy. 2. Confidential, low-cost contraceptive education and services are most likely to increase contraceptive use by teens who are sexually active and therefore decrease teen pregnancy rates. 3. Parents are the biggest influence on teens' decisions to begin or avoid sexual activity. Parents should be encouraged to talk openly and frankly with their teens about their views on sex, contraception, and abstinence. 4. Abstinence first with information on contraception is most effective in reducing teen pregnancy rates. Page Ref: 243

16) The nurse is planning a community adolescent pregnancy prevention program aimed toward parents. Which recommendation should be included in the program in order to be effective? 1. Parents should encourage steady dating. 2. Parents should not allow their son to develop an intense relationship with a girl who is much younger. 3. Rather than embarrassing an adolescent by addressing specific topics related to sex, parents should speak in broad, general terms. 4. Instead of bringing up the topic of sex, parents should allow their children to reach a point where the children initiate the discussion.

Answer: 2 Explanation: 1. Parents need to clearly discourage early dating as well as frequent and steady dating. 2. Parents should take a strong stand against allowing a daughter to date a much older boy; nor should they allow a son to develop an intense relationship with a much younger girl. 3. Parents should be specific in their discussions about sex. 4. Parents need to talk with their children about sex early and often and be specific in the discussions. Page Ref: 244

1) The nurse is preparing a class for expectant fathers. Which information should the nurse include? 1. Siblings adjust readily to the new baby. 2. Sexual activity is safe for normal pregnancy. 3. The expectant mother decides the feeding method. 4. Fathers are expected to be involved in labor and birth.

Answer: 2 Explanation: 1. Siblings often have difficulty adapting to the arrival of a new baby. Regression is often seen in siblings' behaviors. 2. During a normal pregnancy, sexual activity is safe for both mother and baby. 3. Often, the father wants input into the feeding method. 4. In some cultures, labor and birth are only for women, and it is inappropriate for fathers to be involved with the labor and birth. Page Ref: 213

11) The nurse is conducting an early pregnancy class for a group of pregnant women. Which statement indicates a need for further education about the placenta? 1. "It develops and grows larger until about 20 weeks' gestation." 2. "It floats in the amniotic sac and filters waste products from the fetus." 3. "It creates hormones and enzymes that are necessary during pregnancy." 4. "It ages and becomes less permeable during the last month of pregnancy."

Answer: 2 Explanation: 1. The chorionic villi of the placenta become more differentiated with time. The placenta grows in size until about 20 weeks. After this point, the placenta thickens but does not increase in size. 2. The placenta is attached to the uterine wall, and does not float in the amniotic sac. One function of the placenta is to filter metabolic waste products from the baby's blood so that they can be excreted by the mother. 3. The placenta creates glycogen, cholesterol, hormones such as human chorionic gonadotropin (hCG), and enzymes such as sulfatase and insulinase. 4. The placenta is designed to last for 40 weeks, the average length of human gestation. The permeability to nutrients and oxygen begins to decrease starting at about 36 weeks as a part of the aging of the placenta.

1) A nurse is teaching a classroom of teenage girls about the female reproductive system. After teaching, the nurse asks the students to describe the release of an ovum during ovulation. Which response indicates that teaching has been effective? 1. "Ovulation is when the uterus releases an unfertilized egg or ovum." 2. "During ovulation, an egg is released from the ovary and enters the fallopian tube." 3. "The endometrium is where the eggs are formed and released into the fallopian tube." 4. "Around the middle of the menstrual cycle, one of the fallopian tubes releases an egg."

Answer: 2 Explanation: 1. The egg is formed in the ovary and travels by way of the fallopian tube to the uterus. 2. The egg is formed in the ovary and once released, it enters the fallopian tube. 3. The uterine endometrium is the site of implantation of a fertilized egg. 4. The egg is formed in the ovary and then released near the fimbria of the fallopian tube.

9) A pregnant woman is having a nipple-stimulated contraction stress test. Which result indicates hyperstimulation? 1. There are more than five fetal movements in a 10-minute period. 2. There are more than three uterine contractions in a 6-minute period. 3. The fetal heart rate accelerates when contractions last up to 60 seconds. 4. The fetal heart rate decelerates when three contractions occur within a 10-minute period.

Answer: 2 Explanation: 1. The fetal movement is considered a negative contraction stress test. 2. An equivocal or suspicious test has nonpersistent late decelerations or decelerations associated with hyperstimulation (contraction frequency of every 2 minutes or duration lasting longer than 90 seconds). When this test result occurs, more information is needed. 3. The acceleration of the heart rate is considered a negative contraction stress test. 4. Decelerations are considered a positive contraction stress test. Page Ref: 260 breast stimulation is natural method of obtaining oxytocin if 3 spontaneous contractions of good quality and lasting 40-60 seconds occur within a 10 minute window, the results are evaluated and the test is concluded. if no contractions occur or they are insufficient for interpretation, oxytocin is administered or additional breast stimulation is performed.

12) A client who has experienced a fetal death in utero at 20 weeks asks what her baby will look like when it is delivered. How should the nurse respond to this client? 1. "The genitals of the baby will be ambiguous." 2. "Your baby will be covered in fine hair called lanugo." 3. "Your child will have arm and leg buds, but not fully formed limbs." 4. "A white, cheesy substance called vernix caseosa will be on the skin."

Answer: 2 Explanation: 1. The genitals are apparent by 12 weeks after fertilization. This fetus would have had specifically male or female genitals 8 weeks ago. 2. Downy fine hair called lanugo covers a 20-week fetus. 3. Limb buds have developed by 35 days postfertilization. At 20 weeks, the fetus will have well-developed well-differentiated arms and legs. 4. Vernix caseosa forms at about 24 weeks. This fetus is only 20 weeks and will not have vernix.

7) The school nurse is teaching a health class to middle school children. Which structure should the nurse explain as secreting follicle-stimulating hormone (FSH) and luteinizing hormone (LH)? 1. Hypothalamus 2. Anterior pituitary 3. Posterior pituitary 4. Ovaries and testes

Answer: 2 Explanation: 1. The hypothalamus secretes gonadotropin-releasing hormone to the pituitary gland in response to signals from the central nervous system. 2. The anterior pituitary secretes FSH and LH, which are primarily responsible for maturation of the ovarian follicle. 3. The posterior pituitary gland secretes oxytocin and antidiuretic hormone. 4. The ovaries secrete the female hormones estrogen and progesterone, and the testes secrete testosterone.

5) A pregnant adolescent asks about fundal height. What should the nurse say that incorporates the location of the uterine fundus? 1. It is the lower third area of the uterus. 2. It is at the uppermost (dome-shaped top) portion of the uterus. 3. It is the elongated portion of the uterus where the fallopian tubes enter. 4. It is the portion of the uterus that lies between the internal cervical os and the endometrial cavity

Answer: 2 Explanation: 1. The lower third of the uterus is called the cervix or neck. 2. The rounded, uppermost (dome-shaped top) portion of the uterus that extends above the points of attachment of the fallopian tubes is called the fundus. 3. The elongated portion where the fallopian tubes enter the uterus is called the cornua. 4. The portion of the uterus between the internal cervical os and the endometrial cavity is called the isthmus.

9) A client in labor who is receiving a continuous infusion of a local anesthetic through an epidural catheter asks if ear ringing is supposed to occur. What is the most likely cause of the client's complaint? 1. Dehydration 2. Hypotension 3. Allergic reaction 4. Local anesthetic toxicity

Answer: 4 Explanation: 1. Sensation of ringing in the ears is not an allergic reaction. 2. Although maternal hypotension is associated with epidural anesthesia, a sensation of ringing in the ears is associated with local anesthetic toxicity. 3. Sensation of ringing in the ears is not associated with hydration status. 4. Sensation of ringing in the ears is associated with local anesthetic toxicity.

7) A client in the first trimester of pregnancy is experiencing nausea. What should the nurse suggest the client do to relieve the nausea? 1. Eat spicy foods. 2. Eat small, frequent meals. 3. Avoid carbonated beverages. 4. Wait to eat for 2 hours in the morning.

Answer: 2 Explanation: 1. The nausea of pregnancy can be exacerbated by ketosis, fatigue, and certain foods, such as those containing caffeine or spices. 2. Avoiding severe hunger by eating small, frequent meals throughout the day can help to prevent or decrease the severity of the nausea. 3. Carbonated beverages might be helpful in decreasing nausea. 4. Eating dry carbohydrates prior to rising each day can help to prevent or decrease the severity of the nausea. Page Ref: 208

5) What should the nurse do when assisting a pregnant client who is having an abdominal ultrasound to determine fetal age? 1. Has the woman empty her bladder before the test begins 2. Assists the woman into a supine position on the examining table 3. Asks the woman to sign an operative consent form prior to the procedure 4. Instructs the woman to eat a fat-free meal 2 hours before the scheduled test time

Answer: 2 Explanation: 1. The recommendation is that the client should have a full bladder to help elevate the uterus out of the pelvic cavity for better visualization. 2. Clients are placed in a supine position on the table. 3. Abdominal ultrasounds are not invasive procedures and do not require a consent form. 4. Dietary intake is not relevant to the ultrasound. Page Ref: 254

13) During a prenatal examination the healthcare provider is going to estimate the adequacy of the client's pelvis for birth. Which measurement will the physician perform vaginally? 1. True conjugate 2. Diagonal conjugate 3. Obstetric conjugate 4. Transverse outlet diameter

Answer: 2 Explanation: 1. The true conjugate is a measurement of the pelvic inlet and cannot be directly measured. 2. The diagonal conjugate is measured from the lower edge of the symphysis to the sacral promontory. 3. The obstetric conjugate is a measurement of the pelvic inlet and cannot be directly measured. 4. The transverse outlet diameter is measured externally.

14) The nurse is working with a prenatal client. Which statement indicates that additional teaching about prenatal screening tests is necessary? 1. "My blood will be checked for hemoglobin level." 2. "Because I am married, I won't have the STI screening." 3. "My vagina will be cultured at 36 weeks for group B strep." 4. "I will have Rh testing, even though this is my first pregnancy."

Answer: 2 Explanation: 1. This is a true statement. All women will have their hemoglobin assessed. 2. All women should be screened for syphilis, gonorrhea, and hepatitis B. 3. This is a true statement. Women are tested for group B strep to prevent neonatal infection. 4. This is a true statement. All clients are screened for blood type, Rh factor, and Rh antibodies, regardless of how many previous pregnancies (if any) they have had. Page Ref: 187

16) The nurse is completing an assessment for a prenatal visit. Which client statement indicates that further teaching is necessary? 1. "Now that I've felt fetal movement, I should feel movement regularly." 2. "Because I'm in my third trimester, I should return to the clinic in a month." 3. "Alcohol is possibly harmful to my baby, even at the end of my pregnancy." 4. "Before I take any over-the-counter medications, I should contact my doctor."

Answer: 2 Explanation: 1. This is a true statement. Once fetal movement is perceived, it should be felt regularly. Initially, this might not be every day, but in the third trimester, fetal movement should be noticeable several times per day. 2. This statement is incorrect because prenatal visits during the third trimester are every 2 weeks from 26 to 36 weeks, and every week from 36 weeks to delivery. 3. This is a true statement. Alcohol should be avoided throughout pregnancy and lactation. 4. This is a true statement. Regardless of the gestational age, over-the-counter medications can have deleterious effects on the mother or baby; thus, it is important for a pregnant woman to consult her healthcare provider prior to taking any over-the-counter medications throughout the pregnancy. Page Ref: 189

12) A pregnant client in the 21st week of pregnancy is planning a vacation with family and asks which method of travel she should use. How should the nurse respond? 1. "Travel by bus." 2. "Fly on an airplane." 3. "Take an automobile." 4. "Do not travel this late in the pregnancy."

Answer: 2 Explanation: 1. Traveling by bus is similar to traveling by automobile, which does not allow for frequent enough ambulation. 2. As pregnancy progresses, long-distance trips are best taken by plane. 3. Automobile travel does not allow for frequent enough ambulation. 4. It is not necessary to cease travel altogether. Page Ref: 210

12) A woman who is at 12 weeks' gestation asks the nurse if she can undergo chorionic villus sampling (CVS) testing in order to determine whether her baby has a neural tube defect. Which response is best? 1. "No, because CVS testing is not performed until after 20 weeks' gestation." 2. "No, because CVS testing alone at any stage cannot detect neural tube defects." 3. "Yes, at 12 weeks' gestation, CVS can be used to diagnose a neural tube defect." 4. "Yes, at 12 weeks' gestation, CVS is combined with amniocentesis to diagnose neural tube defects."

Answer: 2 Explanation: 1. While CVS is typically performed between 10 and 13 weeks' gestation, this test cannot detect neural tube defects. 2. Because CVS testing is performed so early in the pregnancy, it cannot detect neural tube defects. 3. CVS is typically performed between 10 and 13 weeks' gestation; however, CVS does not detect neural tube defects. 4. CVS is typically performed between 10 and 13 weeks' gestation; however, amniocentesis is not performed until 15 weeks' gestation. Page Ref: 263 CVS -performed to dx genetic, metabolic, and DNA studies

13) A woman who is at 15 weeks' gestation received normal chorionic villus sampling (CVS) results and abnormal quadruple screen test results. For detection of congenital anomalies, which test should the nurse expect the woman to be offered next? 1. Ultrasound 2. Amniocentesis 3. Non-stress test (NST) 4. Contraction stress test (CST)

Answer: 2 Explanation: 1. While ultrasound has many uses, it is not useful in the diagnosis of congenital anomalies. 2. Women who have a normal CVS and an abnormal quadruple screen test would be offered amniocentesis to screen for congenital anomalies. 3. The non-stress test is used to assess fetal status. 4. The contraction stress test is used to assess fetal status. Page Ref: 263 The quad screen is a maternal blood screening test, testing for AFP , hCG , Estriol, and Inhibin-A.

11) A client in labor is concerned about needing a cesarean section and being asleep during the birth of her baby. Which nursing response is most appropriate? 1. "Your anesthesia provider will require that you go to sleep for surgery." 2. "If a cesarean section is needed, that does not necessarily mean you will need to go to sleep for surgery." 3. "We will do our best to make sure you deliver vaginally, so you do not need to have a cesarean section." 4. "If you need a cesarean section, the anesthesia provider will awaken you as soon as possible after delivery so that you can see your baby quickly."

Answer: 2 Explanation: 1. General anesthesia may be needed for cesarean birth and for surgical intervention with some complications. In modern obstetrics, spinal anesthesia is often administered for delivery via cesarean section. 2. While general anesthesia may be needed for cesarean birth and for surgical intervention with some complications, in modern obstetrics, general anesthesia is not used for all obstetric births. 3. Reassuring the client in this manner does not address the erroneous belief that general anesthesia is mandatory for women undergoing cesarean section. 4. Reassuring the client in this manner does not address the erroneous belief that general anesthesia is mandatory for women undergoing cesarean section.

8) A client in labor who is requesting an epidural asks if the baby will be harmed. How should the nurse respond? 1. "Epidural anesthesia is very safe and there are no potential side effects that can affect your baby." 2. "We'll monitor your baby continuously so we can recognize and treat any changes that may be related to the epidural." 3. "We'll assess your blood pressure every 15 minutes to make sure the epidural is not having any negative effects on your baby." 4. "Before your epidural is placed, we'll administer IV fluid to you in order to prevent the epidural from causing you problems."

Answer: 2 Explanation: 1. While proficient administration and monitoring of epidural anesthesia allow for a high degree of safety with this technique, maternal hypotension associated with epidural anesthesia may produce harmful fetal effects. 2. Continuous electronic fetal monitoring to assess fetal status is indicated in the care of pregnant clients who receive epidural anesthesia and allows for a more direct assessment of fetal status than does frequent monitoring of maternal blood pressure and pulse, which is also indicated in the care of this client. 3. While frequent monitoring of maternal blood pressure and pulse is indicated in the care of a client who receives an epidural during labor, continuous electronic fetal monitoring is also indicated for assessment of fetal status and allows for a more direct fetal assessment. 4. While administration of a bolus of IV fluid is indicated in preparation for epidural placement and reduces the risk for maternal hypotension, this intervention neither guarantees the prevention of related complications nor allows for assessment of fetal status.

14) A client at 39 weeks' gestation is having a cesarean birth with general anesthesia. Which potential challenge is most relevant to the anesthesia care of this client? 1. Broad ligament hematoma 2. Difficulty with maternal intubation 3. Hypotension due to the intense blockade of sympathetic fibers 4. Fetal depression that is inversely proportional to maternal anesthetic depth and duration

Answer: 2 Explanation: 1. Broad ligament hematoma is a complication associated with pudendal blockade. 2. Difficulty with maternal intubation is a primary challenge of general anesthesia care for pregnant clients. 3. Regional anesthesia, including epidural anesthesia, is associated with an intense blockade of sympathetic fibers that results in a high incidence of hypotension. 4. Fetal depression associated with general anesthesia is directly proportional to maternal anesthetic depth and duration.

16) A pregnant client is hesitant to have nuchal translucency testing. What should the nurse explain as being the advantages of having this test? Select all that apply. 1. It has a high false-positive rate. 2. It is performed early in the pregnancy. 3. There is no risk of spontaneous abortion. 4. It provides reassurance of the fetus's development. 5. It accurately detects 90% of Down syndrome fetuses.

Answer: 2, 3, 4, 5 Explanation: 1. A high false-positive rate would be a disadvantage of this test. 2. Nuchal translucency testing can be performed in the first trimester to determine if a fetus is at risk for chromosomal disorders. 3. Since it is noninvasive, there is no risk of spontaneous abortion. 4. A normal result can provide reassurance to the woman that her baby most likely does not have a chromosomal disorder. 5. Nuchal translucency testing accurately detects 90% of fetuses with Down syndrome. Page Ref: 256

16) The nurse is preparing information about reproduction for high school students. What should the nurse include about the production of sperm? Select all that apply. 1. Diploid cells are formed. 2. Sperm cells develop through meiotic division. 3. Four spermatids are formed during the second meiotic division. 4. Spermatocytes contain either a double-structured X or Y sex chromosome. 5. Two secondary spermatocytes contain 22 double-structured autosomal chromosomes.

Answer: 2, 3, 4, 5 Explanation: 1. The formation of diploid cells occurs during mitosis. 2. Meiosis occurs during gametogenesis, the process by which sperm are produced. 3. During the second meiotic division, they divide to form four spermatids, each with the haploid number of chromosomes. 4. Spermatocytes contain either a double-structured X sex chromosome or a double-structured Y sex chromosome. 5. During the first meiotic division, the spermatogonium replicates and forms two cells called secondary spermatocytes, each of which contains 22 double-structured autosomal chromosomes.

20) A pregnant client who restricted the intake of fat, protein, and sugar to prevent a large weight gain delivers a small-for-gestational-age fetus. What long-term health problems is this child at risk for developing as an adult? Select all that apply. 1. Arthritis 2. Diabetes 3. Hypertension 4. Cystic fibrosis 5. Coronary heart disease

Answer: 2, 3, 5 Explanation: 1. Arthritis is not specifically linked to nutritional deficiencies in utero. 2. Poor maternal nutrition may also predispose babies who were small or disproportionate at birth to the development of adult diabetes. 3. Poor maternal nutrition may also predispose babies who were small or disproportionate at birth to the development of adult hypertension. 4. Cystic fibrosis is a genetic disease seen in pediatric clients. 5. Poor maternal nutrition may also predispose babies who were small or disproportionate at birth to the development of adult coronary heart disease. Page Ref: 153

15) During a home visit the nurse observes a pregnant client assess fetal activity. Which observations indicate that the client understands the correct process for this count? Select all that apply. 1. Sits in a chair 2. Assumes a side-lying position 3. Counts the same time every day 4. Watches television while counting 5. Begins counting 1 hour after a meal

Answer: 2, 3, 5 Explanation: 1. The client should be in a side-lying position when assessing fetal activity. 2. A side-lying position is the position for assessing fetal activity. 3. The count should be conducted the same time every day. 4. The environment should be quiet during the count. 5. The count should occur about 1 hour after a meal. Page Ref: 253

18) A client's amniocentesis results indicate that the fetus is at risk for respiratory distress. What testing values support this clinical decision? Select all that apply. 1. Amniotic glucose level 50 mg/dL 2. Phosphatidylglycerol (PG) negative 3. Lecithin/sphingomyelin (L/S) ratio 1:6 4. Amniotic fluid red blood cell count 5 mg/dL 5. Lamellar body counts (LBCs) 5000/counts/mcL

Answer: 2, 3, 5 Explanation: 1. There is no information to support the use of amniotic fluid glucose level to predict respiratory functioning. 2. The absence of phosphatidylglycerol (PG) indicates the fetal lungs are not mature. PG is a phospholipid in surfactant. 3. Lecithin and sphingomyelin are two components of surfactant. Early in pregnancy, the sphingomyelin concentration in amniotic fluid is greater than the concentration of lecithin, and so the L/S ratio is low (lecithin levels are low and sphingomyelin levels are high). This can result in the development of respiratory distress syndrome (RDS). 4. There is no information to support the use of amniotic fluid red blood cell counts to predict respiratory functioning. 5. When the LBC is 30,000 to 40,000 counts/mcL, probable lung maturity is assumed. Page Ref: 263

The nurse suspects that a pregnant client is experiencing effects to the gastrointestinal system because of elevated progesterone levels. What did the nurse assess to make this clinical determination? Select all that apply. 1. Nausea 2. Bloating 3. Diarrhea 4. Vomiting 5. Constipation

Answer: 2, 5 Explanation: 1. Nausea is common during the first trimester and may result from several factors, including elevated human chorionic gonadotropin (hCG) levels, relaxation of the smooth muscle of the stomach, and changed carbohydrate metabolism. 2. Elevated progesterone levels cause smooth muscle relaxation, resulting in delayed gastric emptying and decreased peristalsis. As a result the pregnant woman may complain of bloating. 3. Diarrhea is not an expected effect of pregnancy on the gastrointestinal system. 4. Vomiting is common during the first trimester and may result from several factors, including elevated human chorionic gonadotropin (hCG) levels, relaxation of the smooth muscle of the stomach, and changed carbohydrate metabolism 5. Elevated progesterone levels cause smooth muscle relaxation, resulting in delayed gastric emptying and decreased peristalsis. As a result the pregnant woman may complain of constipation.

4) Which client should the nurse identify as being a multipara? 1. A client at 28 weeks' gestation with no previous pregnancies 2. A client at 32 weeks' gestation who previously delivered one term infant 3. A client at 13 weeks' gestation who previously delivered two term infants 4. A client at 34 weeks' gestation who previously had one spontaneous abortion

Answer: 3 Explanation: 1. A woman who has had no births at more than 20 weeks' gestation is considered a nullipara. 2. A woman who has had one birth at more than 20 weeks' gestation, regardless of whether the infant was born alive or dead, is considered a primipara. 3. A woman who has had two or more births at more than 20 weeks' gestation is considered a multipara. 4. A woman who has had no births at more than 20 weeks' gestation is considered a nullipara. Page Ref: 171

2) A woman at 28 weeks' gestation reports not feeling the baby move for over 30 minutes. How should the nurse respond first? 1. "When did you eat last?" 2. "Have you been smoking?" 3. "Your baby might be asleep." 4. "You need to go to the emergency room immediately for further evaluation."

Answer: 3 Explanation: 1. After meals, an infant typically is active and moving. 2. Smoking typically will stimulate the infant. 3. Lack of fetal activity for 30 minutes typically is insignificant and means only that the infant is sleeping. The mother should continue to observe for fetal movements over the next 2.5 hours. If a lack of fetal movements continues, she should contact the healthcare provider. 4. The mother would need to come to be examined if there had been no fetal activity for several hours. Page Ref: 253

A client in the prenatal clinic believes she is pregnant because she has not menstruated for 3 months, and her breasts are getting bigger. What response by the nurse is best? 1. "Lack of menses and breast enlargement are presumptive signs of pregnancy." 2. "The changes you are describing are definitely indicators that you are pregnant." 3. "Lack of menses can be caused by many things. We need to do a pregnancy test." 4. "Breast and menstrual changes are positive signs of pregnancy. Congratulations."

Answer: 3 Explanation: 1. Although this is true, amenorrhea and breast enlargement also can be caused by weight gain and other conditions. A pregnancy test is needed to determine whether the client is pregnant. 2. This statement is false because amenorrhea and breast enlargement are presumptive signs of pregnancy because they can be caused by other conditions. 3. This is a true statement and addresses that these changes could be caused by things other than pregnancy. 4. This statement is false because amenorrhea and breast enlargement are presumptive signs of pregnancy. It is too early to determine if congratulations are in order.

4) A client who is at 10 weeks' gestation is concerned about the amount of saliva that is in her mouth since she seems to be spitting when she talks. How should the nurse respond? 1. "You should avoid astringent mouthwashes and chewing gum." 2. "That's called ptyalism, and it's usually caused by increased salt intake during the second trimester." 3. "Excess salivation commonly occurs during the first trimester, although the cause is unknown." 4. "Let's schedule you for a doctor's appointment, because excessive salivation can signal a complication of pregnancy."

Answer: 3 Explanation: 1. Astringent mouthwashes, chewing gum, and sucking hard candy may help relieve the bitter taste that often accompanies ptyalism. 2. Ptyalism, which is excess production of saliva, usually occurs during the first trimester and the cause is unknown. 3. Ptyalism, which is excess production of saliva, commonly occurs during the first trimester and the cause is unknown. 4. Excess salivation, also called ptyalism, is a normal occurrence in women during the first trimester. Page Ref: 205

13) The nurse is explaining the importance of fetal activity assessment to a pregnant client. What should the nurse do to best reinforce the significance of fetal kick monitoring? 1. Perform daily phone calls to the client at work or home. 2. Ask the client to remember to count the fetal movements. 3. Explain the reason for counting fetal movement to the client. 4. Review the client's written record of fetal movement at each visit.

Answer: 3 Explanation: 1. Daily phone calls would take emphasis away from the importance of the client's counting of fetal movement. 2. Writing down the count is more accurate than the client's simply remembering. When the nurse examines the written record the client has kept, it reinforces the importance of the record and improves the likelihood of continued record keeping. 3. Many healthcare providers encourage pregnant women to monitor their unborn child's well-being by regularly assessing fetal activity beginning at 28 weeks' gestation. Vigorous activity generally provides reassurance of fetal well-being, but a marked decrease in activity or cessation of movement may indicate a problem that needs immediate evaluation. 4. Knowing the reasons for the counting will increase understanding of the process but will not reinforce the significance of the task. Page Ref: 207

17) The nurse is preparing a prenatal class about infant feeding methods to include maternal nutritional requirements for breastfeeding and formula-feeding. What statement should the nurse include? 1. "Formula-feeding mothers need a high protein intake to avoid fatigue." 2. "Producing breast milk requires calories, but any source of food is fine." 3. "Breastfeeding requires a continued high intake of protein and calcium." 4. "Formula-feeding mothers should protect their health with a lot of calcium."

Answer: 3 Explanation: 1. Formula-feeding moms do not need additional nutrients. 2. Although any food source would provide the additional calories, these needs are best met by using increased protein intake as the source for the required additional calories to support milk production. 3. Lactation requires calories, along with increased protein and calcium intake. 4. Formula-feeding mothers do not need additional nutrients.

13) At her first prenatal visit, the client states, "I'm 5 weeks pregnant now and I would like to hear my baby's heartbeat today." How should the nurse respond? 1. Anticipate that the client will be scheduled for Doppler ultrasound. 2. Prepare to assist with auscultation of the fetal heartbeat using a fetoscope. 3 Explain to the client that the fetal heartbeat is not yet detectable at 5 weeks' gestation. 4. Explain to the client that the fetal heart does not begin to beat until approximately 7 weeks' gestation.

Answer: 3 Explanation: 1. Generally, fetal heart tones cannot be heard until approximately the 8th week of pregnancy by ultrasound Doppler device. 2. Fetal heart tones can be identified through use of a fetoscope at approximately the 20th week of pregnancy. 3. While the tubular heart begins to form during the 3rd week, fetal heart tones generally are not detectable until at least 7 weeks' gestation. 4. The fetal heart begins to beat by the 4th week of gestation.

9) Which statement by a client pregnant with twins would indicate that teaching was effective? 1. "Identical twins can be the same or different sex." 2. "Identical twins occur more frequently than fraternal twins." 3. "Congenital abnormalities are more prevalent in identical twins." 4. "Because of their birth relationship, fraternal twins are more similar to each other than if they had been born singly."

Answer: 3 Explanation: 1. Identical or monozygotic twins have identical chromosomal structures and therefore are always the same sex. 2. Dizygotic, or fraternal, twins occur more frequently than do monozygotic twins. 3. Due to variations in the timing of the splitting of the embryo, congenital abnormalities are more common in monozygotic twins. 4. Fraternal twins are not more similar to each other than if they had been born singly.

2) The pregnant client's prenatal record indicates that she is a gravida 4 para 2022. How should the nurse interpret these data about the client? 1. Is pro-abortion 2. Has four living children 3. Delivered two term infants 4. Delivered two infants preterm

Answer: 3 Explanation: 1. In the four-digit number, the third digit indicates the number of abortions the client has experienced. Because abortion may be spontaneous or therapeutic, this number does not necessarily reflect a woman's stance on surgical abortion. 2. In the four-digit number, the fourth number indicates the number of living children, which is 2. 3. In the four-digit number, the first digit indicates the number of term infants born, which is 2. 4. In the four-digit number, the second digit indicates the number of preterm births, so the client has had no preterm births. Page Ref: 171

2) The nurse is preparing an antenatal nutrition class for pregnant women. Which material should be included in the teaching? 1. During pregnancy, consumption of oily fish should be avoided. 2. Dietary protein can only be obtained through consuming dairy, meat, and eggs. 3. Nutritional iodine requirements generally can be met through intake of iodized salt. 4. Iron absorption is generally higher for vegetable products than for animal products.

Answer: 3 Explanation: 1. Oily fish provide the best source of docosahexaenoic acid (DHA), which may reduce the risk of preterm birth, preeclampsia, low birth weight, and enhance fetal and infant brain development. 2. Excluding dairy, meat, and eggs, adequate dietary protein can be obtained by consuming a varied diet with adequate caloric intake and plant-based proteins. 3. Intake of iodized salt generally provides the recommended intake of iodine. 4. Iron absorption is generally higher for animal products than for vegetable products.

13) The nurse is working in a teen pregnancy clinic. In order to give the pregnant adolescent a role in her prenatal care, what should the nurse encourage the teen to do? 1. Choose the schedule of her prenatal visits 2. Decide if she wants her labor to be induced 3. Measure and record her weight at each visit 4. Choose the type of prenatal vitamin she takes

Answer: 3 Explanation: 1. Prenatal visit schedules are set to detect developing complications of pregnancy. 2. Induction of labor is a medical decision and should not be taken lightly. 3. Having the client weigh herself and record her weight provides her with information that indicates she is growing a healthy fetus. 4. Prenatal vitamins are prescribed by the certified nurse-midwife or the physician. Many formulations exist, and some might not be indicated for this client due to her nutritional practices and lab results. In addition, if the client is a member of a health maintenance organization, only certain medications (including prenatal vitamins) are accepted for coverage. Page Ref: 241

5) The client has delivered her first child at 39 weeks. How should the nurse document this type of delivery? 1. Preterm 2. Postterm 3. Full Term 4. Near term

Answer: 3 Explanation: 1. Preterm deliveries are those that occur prior to 36 completed weeks' gestation. 2. Postterm applies to births that occur after 42 weeks' gestation. 3. Full-term births occur between 39 weeks 0 days and 40 weeks 6 days. 4. Near term is not terminology used to describe birth. Page Ref: 171

6) A 32-year-old primipara who is at 8 weeks' gestation asks if she should expect any breathing changes as the pregnancy progresses. How should the nurse respond? 1. "By the third trimester, you will no longer feel as though you're short of breath." 2. "You may experience shortness of breath due to stretching of the round ligament." 3. "If you develop shortness of breath, it should improve in the last few weeks of your pregnancy, as lightening occurs." 4. "Shortness of breath is an abnormal finding during any stage of pregnancy, and it is considered a serious complication."

Answer: 3 Explanation: 1. Shortness of breath occurs as the uterus rises into the abdomen and causes pressure on the diaphragm. This problem worsens in the last trimester. 2. Round ligament stretching causes a "grabbing" sensation in the lower abdomen and inguinal area. 3. The primigravida experiences considerable relief from shortness of breath in the last few weeks of pregnancy, when lightening occurs and the fetus and uterus move down in the pelvis. 4. Because of decreased vital capacity from pressure of the enlarging uterus on the diaphragm, shortness of breath is a common problem of pregnancy. Page Ref: 207

9) A client in the third trimester of pregnancy reports working 8 hours a day as a cashier and stands when at work. What response by the nurse is the most appropriate? 1. "No problem. Your baby will be fine." 2. "Do you get regular breaks for eating?" 3. "Your risk of preterm labor is higher." 4. "Standing might increase ankle swelling."

Answer: 3 Explanation: 1. Standing more than 5 hours a day increases the risk of preterm labor. To be therapeutic in communication, avoid false reassurance. 2. Although breaks for eating, drinking, and toileting are important for pregnant employees, it is more important to tell the client about the increased risk of preterm labor. 3. Pregnant women who stand for more than 3 hours a day have an increased risk of preterm labor. Because preterm labor can put the infant's life at risk, this statement would be the highest priority. 4. Although this is true, it is less important than teaching the client about the risks of preterm labor when standing more than 3 hours a day. Page Ref: 210

While auscultating fetal heart tones, a client who is at 37 weeks' gestation and is in the supine position is experiencing dizziness, lightheadedness, and clammy skin. Which nursing action is the most appropriate? 1. Administer supplemental oxygen. 2. Help the client turn onto her right side. 3. Place a wedge beneath the client's right hip. 4. Prepare for administration of packed red blood cells (PRBCs).

Answer: 3 Explanation: 1. Supplemental oxygen is not required for supine hypotension syndrome. 2. Positioning the client on her right side would likely exacerbate the reduction in right atrial blood flow. 3. The client is verbalizing symptoms consistent with supine hypotension syndrome, in which compression of the vena cava by the uterus reduces right atrial blood flow. Signs and symptoms include decreased blood pressure, dizziness, pallor, and clamminess. Appropriate interventions include having the woman lie on her left side, or placing a pillow or wedge under her right hip as she lies in a supine position. 4. Packed red blood cell infusion is not required to treat supine hypotension syndrome.

12) When preparing nutritional instruction, which pregnant client should the nurse consider the highest priority? 1. 40-year-old gravida 2 2. 35-year-old gravida 4 3. 15-year-old nulligravida 4. 22-year-old primigravida

Answer: 3 Explanation: 1. The 40-year-old client has completed her growth cycle, and her body can focus on diverting the nutritional needs to the fetus. 2. The 35-year-old client has completed her growth cycle, and her body can focus on diverting the nutritional needs to the fetus. 3. Adolescent clients typically are still in their own growth cycle. Suddenly, they have to supply nutrition for themselves and the fetus. This places them at greatest risk for malnutrition. 4. The 22-year-old client has completed her growth cycle, and her body can focus on diverting the nutritional needs to the fetus.

6) If only a small volume of sperm is discharged into the vagina, an insufficient amount of enzymes might be released when sperm encounters the ovum. What would be the result for pregnancy? 1. The block to polyspermy (cortical reaction) would not occur. 2. The fertilized ovum would be unable to implant in the uterus. 3. Sperm would be unable to penetrate the zona pellucida of the ovum. 4. Peristalsis of the fallopian tube would decrease, making it difficult for the ovum to enter the uterus.

Answer: 3 Explanation: 1. The answer choice of the block to polyspermy (cortical reaction) not occurring is incorrect because it is mediated by release of materials from cortical granules below the ovum's surface and not the result of low sperm count. 2. With a low sperm count it is unlikely that the ovum would be fertilized. 3. Sperm would be unable to penetrate the zona pellucida of the ovum because it takes hundreds of acrosomes (the result of the acrosomal reaction) to rupture and release enough hyaluronic acid to clear the way for a single sperm to penetrate the ovum's zona pellucida successfully. 4. "Peristalsis of the fallopian tube would decrease, making it difficult for the ovum to enter the uterus" is an incorrect statement.

6) A 20-weeks'-gestation adolescent states that it is important not to have a baby that weighs too much and has been limiting her calories so that her current weight has dropped from 110 pounds to 106 pounds. How should the nurse respond? 1. "You are causing harm to your baby." 2. "You shouldn't be worrying about your figure." 3. "Your baby needs adequate nutrition to develop." 4. "It's okay to want a small baby when you're a teen."

Answer: 3 Explanation: 1. The first role of the nurse is to explain why food is important to the growing fetus, specifying how each food group will help the fetus develop. Next, the nurse must assist the pregnant adolescent to plan foods that she likes to eat from each food group. 2. Pregnant adolescents are just adapting to a new body image created by the changes of puberty when the pregnancy produces rapid and substantial body changes. The desire to maintain a socially desirable figure can lead to nutritional deficits. 3. Teens might not understand the physiology behind the profound body changes of pregnancy. 4. Anticipatory guidance in the body changes of pregnancy will assist the adolescent's adjustment to them. Although many teens are anxious, this teen is expressing a direct nutritional deficit. Page Ref: 241

3) A prenatal client states, "The doctor said he might have to cut my cervix so the baby can get out during delivery." Based upon this statement, what structure should the nurse define when teaching about an episiotomy? 1. Mons pubis 2. Labia majora 3. Perineal body 4. Vaginal vestibule

Answer: 3 Explanation: 1. The mons pubis is a softly rounded mound of subcutaneous fatty tissue that covers the front portion of the symphysis pubis. 2. The labia majora are longitudinal, raised folds of pigmented skin located on either side of the vulvar cleft. 3. The perineal body, which is located between the lower part of the vagina and the anus, is often the site of an episiotomy or lacerations during childbirth. 4. The vaginal vestibule contains the vaginal opening, which is the border between the external and internal genitals.

7) At 32 weeks' gestation, a woman is scheduled for a second non-stress test. Which client response indicates an adequate understanding of this procedure? 1. "I'll have an IV started before the test." 2. "I need to have a full bladder for this test." 3. "I cannot get up and walk around during the test." 4. "I must avoid drinks containing caffeine for 24 hours before the test."

Answer: 3 Explanation: 1. There is no IV needed to administer medications. 2. Clients usually are asked to have their bladders full only for ultrasounds. 3. The purpose of the non-stress test is to determine the results of movement on fetal heart rate. The client will have to lie still on her side during the procedure. 4. Caffeine might cause the infant to be more active and cause the test to go more quickly. Page Ref: 257 non-stress test -accelerations of FHR indicate adequate oxygenation and intact CNS -reactive test shows accelerations of 15 beats/min lasting 15 seconds for a 20 minute window -results = reactive or nonreactive

17) The nurse is preparing materials for couples beginning prenatal care. What information is most important for the nurse to include? 1. The birthing unit was remodeled and redecorated last year. 2. Some of the healthcare providers recommend circumcision for baby boys. 3. There are various types of healthcare providers to support the client through the process. 4. There are different types of rooms for giving birth, each with different equipment.

Answer: 3 Explanation: 1. This information does not help clients understand their options or make decisions. 2. Because not all clients will be having boys, this statement is only helpful to those clients who give birth to males and see a healthcare provider who recommends circumcision. This statement is too narrowly focused to be helpful to all clients. 3. This statement is the most important. The nurse should inform clients what their options are, including the types of healthcare providers available. 4. This statement is too vague to facilitate decision making by the couple. Page Ref: 199

13) Which statement is best to include when teaching a pregnant adolescent about nutritional needs? 1. "You just need to pay attention to what you eat now." 2. "Folic acid intake is the key to having a healthy baby." 3. "It is important eat iron-rich foods like meat every day." 4. "Calcium and milk aren't needed until the third trimester."

Answer: 3 Explanation: 1. This response is too vague to be helpful. Adolescents will need specific information to improve nutrition during pregnancy. 2. Although folic acid is important during pregnancy to prevent neural tube defects, and for lactation, there is not one single nutritional element responsible for having a healthy baby. 3. Adolescents often have an iron intake that is inadequate for pregnancy. Giving specific examples is most helpful when providing nutritional information. 4. Calcium is needed throughout pregnancy and should be consumed daily.

11) The nurse is reviewing nursing documentation related to the care of a client who had an amniocentesis. Which nursing note reflects appropriate client care? 1. Prior to discharge, the client demonstrated vaginal spotting. 2. An Rh-positive client received RhoGAM after the amniocentesis. 3. The client was monitored for 30 minutes after completion of the test. 4. The client reported that she takes insulin before each meal and at bedtime.

Answer: 3 Explanation: 1. Vaginal spotting after the amniocentesis is not an expected finding. A client experiencing vaginal bleeding of any amount after amniocentesis requires additional assessment and should not be sent home. 2. Only Rh-negative clients receive RhoGAM after amniocentesis. The Rh-positive client should not ever receive RhoGAM. 3. Twenty to 30 minutes of fetal monitoring is performed to assess for fetal well-being and to rule out injury of the fetus or placenta during the examination. 4. Whether or not a client takes insulin has nothing to do with amniocentesis. This answer does not relate to the question asked. Page Ref: 263

14) A 23-year-old client who is at 10 weeks' gestation with a first pregnancy expresses worry over feeling no sexual desire for her spouse and asks if this is normal. How should the nurse respond? 1. "That's unusual. Throughout a healthy pregnancy, sexual desire usually increases with each trimester." 2. "That's unusual. Usually, there are minimal changes in sexual desire throughout a healthy pregnancy." 3. "That sounds normal. In many cases, sexual desire decreases in the first trimester, but it increases again during the second trimester." 4. "That sounds normal. During the first trimester, sexual desire often decreases; however, by the third trimester, sexual desire is usually greater than before pregnancy."

Answer: 3 Explanation: 1. While each woman may be different, sexual desire often decreases during the first trimester, increases during the second trimester, and then decreases again during the third trimester. 2. While each woman may be different, sexual desire often decreases during the first trimester, increases during the second trimester, and then decreases again during the third trimester. 3. During the first trimester, factors such as fatigue, nausea, vomiting, and breast tenderness may decrease desire for sexual activity. During the second trimester, as these discomforts lessen and pelvic vascular congestion increases, the woman may experience greater sexual satisfaction than before pregnancy. 4. While each woman may be different, sexual desire often decreases during the first trimester, increases during the second trimester, and then decreases again during the third trimester. Page Ref: 214

15) A postpartum client who received spinal anesthesia for the delivery has not voided for 5 hours and is concerned about nerve damage. How should the nurse respond about this concern? 1. "Spinal anesthesia can sometimes cause nerve damage." 2. "You are probably dehydrated. Please increase your water intake." 3. "It may be several hours before you're able to control your urination." 4. "You should be able to control your bladder by now. I'll ask the anesthesia provider to visit with you."

Answer: 3 Explanation: 1. Although nerve damage is a rare occurrence in relation to spinal anesthesia, there are no objective data to suggest that this woman has experienced nerve damage. Restoration of bladder control may take 8 to 12 hours following a spinal anesthetic. 2. There are no data to suggest the woman is dehydrated. Rather, she is likely demonstrating a common side effect of spinal anesthesia. Restoration of bladder control may take 8 to 12 hours following a spinal anesthetic. 3. Restoration of bladder control may take 8 to 12 hours following a spinal anesthetic. 4. Restoration of bladder control may take 8 to 12 hours following a spinal anesthetic. Page Ref: 400

The nurse is preparing an educational program on the different types of pregnancy tests. What should the nurse include about the β-subunit radioimmunoassay (RIA) test? Select all that apply. 1. It causes a color change. 2. It takes 2 to 3 hours to perform. 3. It is able to detect trophoblastic disease. 4. It is able to detect an ectopic pregnancy. 5. It is the same as an over-the-counter test

Answer: 3, 4 Explanation: 1. Enzyme-linked immunosorbent assay (ELISA) uses a substance that results in a color change after binding. 2. Fluoroimmunoassay (FIA) takes about 2 to 3 hours to perform. 3. β-Subunit radioimmunoassay (RIA) uses an antiserum with specificity for the β-subunit of hCG in blood plasma. This test may not only detect pregnancy but also detect trophoblastic disease. 4. β-Subunit radioimmunoassay (RIA) uses an antiserum with specificity for the β-subunit of hCG in blood plasma. This test may not only detect pregnancy but also detect an ectopic pregnancy. 5. Over-the-counter pregnancy tests are ELISA tests. This assay, which may be done on urine or blood, is sensitive and quick. It can detect hCG levels as early as 7 to 9 days after ovulation and conception, which is 5 days before the first missed period.

16) A client in labor is having a pudendal block. For which adverse effects should the nurse assess this client? Select all that apply. 1. Infection 2. Spinal headache 3. Perforated rectum 4. Sciatic nerve trauma 5. Broad ligament hematoma

Answer: 3, 4, 5 Explanation: 1. An infection is not associated with a pudendal block. 2. A spinal headache is not associated with a pudendal block. 3. Disadvantages of the pudendal block include possible perforation of the rectum. 4. Disadvantages of the pudendal block include possible trauma to the sciatic nerve. 5. Disadvantages of the pudendal block include possible broad ligament hematoma. Page Ref: 401

17) The nurse is reviewing the embryonic primary germ layers with a group of students. Which structures should the nurse identify as being formed from the mesoderm? Select all that apply. 1. Nails 2. Liver 3. Spleen 4. Muscles 5. Skeleton

Answer: 3, 4, 5 Explanation: 1. The nails originate from the ectoderm. 2. The liver originates from the endoderm. 3. The spleen originates from the mesoderm. 4. The muscles originate from the mesoderm. 5. The skeleton originates from the mesoderm.

6) A pregnant client is receiving the results of perinatal testing. Which statement indicates that the client understands the test result? 1. "Because my contraction stress test was positive, we know that my baby will tolerate labor well." 2. "The reactive non-stress test means that my baby is not growing because of a lack of oxygen." 3. "My biophysical profile score of 6 points indicates everything being normal and healthy for my baby." 4. "The normal Doppler velocimetry wave result indicates my placenta is getting enough blood to the baby."

Answer: 4 Explanation: 1. A contraction stress test creates mild contractions. The presence of decelerations is termed a positive result and indicates a lack of adequate placental functioning. 2. The non-stress test utilizes external fetal monitoring to assess the fetal heart rate in relationship to fetal movement. When accelerations in the fetal heart rate are associated with fetal movement (a reactive result), the fetus is well oxygenated, and the placenta is functioning well. 3. The biophysical profile score should be 8 (with adequate amniotic fluid) or 10. A score of 6 is abnormal and indicates that further assessment is needed. 4. The Doppler velocimetry test looks at blood flow through the umbilical artery. A normal result indicates there is no vasospasm decreasing blood flow to the placenta; therefore, the baby is getting an adequate blood supply. Page Ref: 256

8) During a non-stress test, the nurse notes that the fetal heart rate decelerates about 15 beats during a period of fetal movement. The decelerations occur twice during the test and last 20 seconds each. What should the nurse expect the outcome of this test will be? 1. A reactive test 2. A negative test 3. An equivocal test 4. A nonreactive test

Answer: 4 Explanation: 1. A reactive stress test has the expected results of an increase in heart rate of 15 beats per minute for 15 seconds or more. 2. Non-stress tests are scored as either reactive or nonreactive. 3. Non-stress tests are scored as either reactive or nonreactive. 4. In a nonreactive stress test, the reactivity criteria are not met. Since this client experienced a deceleration during the test, this is considered nonreactive. Page Ref: 258

3) The school sexual health clinic nurse has female adolescent students waiting to be seen. Which student should be seen first? 1. 17-year-old adolescent with a history of child abuse 2. 14-year-old adolescent whose 17-year-old sister is pregnant 3. 15-year-old adolescent who reports using condoms regularly 4. 16-year-old adolescent who had a chlamydial infection treated 2 weeks ago

Answer: 4 Explanation: 1. Although adolescents with a history of abuse are more likely to become pregnant than are their peers who have not experienced abuse, too little information is given about this client to determine risk for pregnancy. This client is not the top priority. 2. Although this student is at risk for becoming pregnant because her sister is experiencing an adolescent pregnancy, it is not known whether this client is sexually active. This student is not the top priority. 3. Condom use will decrease the risk of becoming pregnant. This client is a low priority. 4. This client is the top priority. Having had a chlamydial infection, a sexually transmitted infection, indicates that the client is sexually active and not using a barrier method of birth control. This client is at risk for pregnancy and another STI. Page Ref: 236

6) The prenatal clinic nurse is designing a new prenatal intake information form for pregnant clients. Which question is the most important to include on this form? 1. What is the name of the baby's father? 2. Where was the father of the baby born? 3. Are you married to the father of the baby? 4. Do genetic diseases run in the family of the baby's father?

Answer: 4 Explanation: 1. Although it is helpful for the nurse to know the name of the father's baby to include him in the prenatal care, this is psychosocial information and much less important than possible genetic diseases that the baby might have inherited. 2. This is not important information for pregnancy. 3. Although the marital status of the client might have cultural significance, this is psychosocial information and much less important than possible genetic diseases that the baby might have inherited. 4. This question has the highest priority because it gets at the physiologic issue of inheritable genetic diseases that might directly impact the baby. Page Ref: 172

An adolescent client reports that her period is late but that her home pregnancy test is negative. Which response is most appropriate? 1. "This means you are not pregnant." 2. "We do not trust home tests. Come to the clinic for a blood test." 3. "Most people do not use the tests correctly. Did you read the instructions?" 4. "You might be pregnant, but it might be too early for your home test to be accurate."

Answer: 4 Explanation: 1. Although this might be true, this is not the best response because the pregnancy may not yet be detectable through use of a urine pregnancy test. 2. This statement is not therapeutically worded. Additionally, this statement is not true because home pregnancy tests are quite simple to use and quite accurate. A clinic pregnancy test is usually a urine test. Blood tests are more invasive and more expensive. 3. This response does not address the issues presented in the client's statement. 4. This is an accurate and appropriate response. Most home pregnancy tests have low false- positive rates, but the false-negative rate is slightly higher. Repeating the test in 1 week is recommended.

8) The nurse is working with male teens whose partners are pregnant. Which situation with a father-to-be requires nursing intervention? 1. The pregnancy does not seem real to him, and he is not sure what he should do to plan for the future. 2. Because his father was not involved in his life, he wants to be actively involved in the life of his child. 3. He is not convinced that he is the father of the baby and does not want his name on the birth certificate. 4. He is the only other person who will be present, although his girlfriend wants her mother to be with her during the birth.

Answer: 4 Explanation: 1. An early pregnancy is often an abstract concept to fathers. It is normal for a teen to be unsure about the future. 2. This might be the case when a young father was raised without his own father, or a lack of involvement with the child might be seen as the norm and desired. 3. Many young fathers are not sure if they are the father of the baby and do not want to be listed on the birth certificate if they are not sure. Often these young men will request paternity testing to either verify or discount their parentage. 4. It is common for pregnant adolescents to want their mothers to accompany them for the labor and birth. Overriding his girlfriend's expressed desire could be an indication that their relationship is abusive. Page Ref: 239

2) The spouse accompanies a pregnant client to a prenatal visit. Which question should the nurse use to determine the amount of anticipatory guidance the spouse will need? 1. "What kind of work do you do?" 2. "How moody has your wife been lately?" 3. "What furniture have you gotten for the baby?" 4. "How are you feeling about becoming a father?"

Answer: 4 Explanation: 1. Asking about vocation does not help determine the amount of anticipatory guidance the spouse will need. 2. Asking about the client's mood does not help determine the amount of anticipatory guidance the spouse will need. 3. Buying furniture does not help determine the amount of anticipatory guidance the spouse will need. 4. Anticipatory guidance of the expectant father is a necessary part of any plan of care. He may need information on a variety of topics about the pregnancy, and the best question to learn the spouse's needs is to ask about his feelings about becoming a father. Page Ref: 195

12) The nurse explains to a pregnant woman that her antepartum assessment will include assessment of clinical pelvimetry. Which client response reflects understanding of the reason for this test? 1. "It will help me understand how big a baby I can have." 2. "It will be used to screen for gestational diabetes." 3. "It will be used to find out whether my baby has a chromosomal abnormality." 4. "It will help tell whether my pelvis is big enough to deliver my baby vaginally."

Answer: 4 Explanation: 1. Clinical pelvimetry is performed to estimate the adequacy of pelvic size for the purpose of vaginal delivery; delivery of larger infants may be accommodated via cesarean section. 2. Screening for maternal gestational diabetes requires some form of glucose screening. 3. Clinical pelvimetry involves estimating the adequacy of pelvic size for facilitating vaginal birth. 4. Clinical pelvimetry is performed to estimate the ease or difficulty associated with vaginal delivery of an infant. Page Ref: 186

11) A pregnant 14-year-old client confides having problems with bulimia nervosa. Which nursing observation best supports the client's statement? 1. The client reports dietary cravings for soil and clay. 2. In terms of food variety and quantity, the client's diet is extremely restrictive. 3. Despite being extremely underweight, the client describes herself as being fat. 4. The client is of normal weight for her height and reports binge eating followed by purging.

Answer: 4 Explanation: 1. Dietary cravings for and consumption of nonnutritive substances is consistent with pica. 2. The dietary intake of individuals with anorexia nervosa is very restrictive in both variety and quantity. 3. Anorexia nervosa is an eating disorder characterized by an extreme fear of weight gain and becoming fat, and it incorporates a self-perception of being overweight even when the individual is extremely underweight. 4. Bulimia is characterized by binge eating and purging, and individuals with bulimia nervosa often maintain normal or near-normal weight for their height.

8) A pregnant client asks about the differences between monozygotic and dizygotic twins. Which should the nurse include during this teaching? 1. Dizygotic twins share one placenta and one chorion. 2. Monozygotic twins are also referred to as "fraternal" twins. 3. Dizygotic twinning occurs less frequently than does monozygotic twinning. 4. Monozygotic twins originate from division of the fertilized ovum at different stages.

Answer: 4 Explanation: 1. Dizygotic twins each have a separate chorion and amnion. 2. Monozygotic twins are also referred to as "identical" twins. 3. Dizygotic twinning occurs more frequently than does monozygotic twinning. 4. Monozygotic twins originate from division of the fertilized ovum at different stages of early development.

A 28-year-old client who is pregnant with her first child reports increased dental caries (cavities) since becoming pregnant. How should the nurse explain the likely cause for this change? 1. "Each woman experiences changes that affect her teeth while she's pregnant." 2. "When a woman is pregnant, her teeth lose calcium and she is more susceptible to getting cavities." 3. "During pregnancy, tooth enamel softens and the woman is more susceptible to getting cavities." 4. "It may be necessary to pay extra attention to dental care while you're pregnant, especially if you're vomiting frequently."

Answer: 4 Explanation: 1. No demonstrable changes occur in the teeth of pregnant women. 2. Calcium is not lost from the teeth during pregnancy. 3. Tooth enamel does not soften during pregnancy. 4. The dental caries that sometimes accompany pregnancy are probably caused by inadequate oral hygiene and dental care, especially if the woman has problems with bleeding gums or nausea and vomiting

2) A client desiring to become pregnant is concerned because the opening of her spouse's penis is below the tip. How should the nurse respond to this client? 1. "This is epispadias and it will cause infertility." 2. "This is hypospadias and it will cause infertility." 3. "This is epispadias and is not likely to impact fertility." 4. "This is hypospadias and is not likely to impact fertility."

Answer: 4 Explanation: 1. Epispadias is the condition where the urethral opening is on the upper aspect of the penis. The client is describing hypospadias, when the urethral opening is on the lower side of the penis. Mild hypospadias, when the urethral opening is on the glans of the penis, does not impact fertility. 2. The client is describing hypospadias, where the urethral opening is on the lower side of the penis. Mild hypospadias, where the urethral opening is on the glans of the penis, does not impact fertility. 3. Epispadias is the condition where the urethral opening is on the upper aspect of the penis. The client is describing hypospadias, when the urethral opening is on the lower side of the penis. Mild hypospadias, when the urethral opening is on the glans of the penis, does not impact fertility. 4. The client is describing hypospadias, which is the urethral opening on the lower aspect of the penis. Mild hypospadias, when the urethral opening is on the glans of the penis, does not impact fertility.

6) The nurse is teaching a client who recently learned that she is pregnant. Which hormone should the nurse instruct as being secreted by the fertilized egg? 1. Estrogen 2. Progesterone 3. Luteinizing hormone 4. Human chorionic gonadotropin (hCG)

Answer: 4 Explanation: 1. Estrogen is an ovarian hormone. 2. Progesterone is an ovarian hormone. 3. Luteinizing hormone is excreted by the anterior pituitary. 4. When the ovum is fertilized and implants in the endometrium, the fertilized egg begins to secrete hCG hormone to maintain the corpus luteum.

1) A 15-year-old primipara at 8 weeks' gestation who is 64 inches tall and weighs 115 pounds asks why she is supposed to gain so much weight. What is the best response by the nurse? 1. "Inadequate weight gain delays lactation after delivery." 2. "It's what your certified nurse-midwife recommended for you." 3. "Weight gain is important to assure that you get enough vitamins." 4. "Gaining 25 to 35 pounds is recommended for healthy fetal growth."

Answer: 4 Explanation: 1. Inadequate weight gain can lead to decreased fetal growth and development. 2. Although this statement might be true, the client has asked a "why" question that should be directly answered. 3. Vitamin intake is related to the types of food consumed, not to caloric intake. Because this client is 15, her diet is probably not optimal, and her intake of empty calories or junk food might make up the majority of her caloric intake. 4. Adolescents who become pregnant less than 4 years after menarche are at risk because of their physiologic and anatomic immaturity. They are more likely than older adolescents to still be growing, which can affect the fetus's development.

2) A client in labor wants to have a medication-free birth. What should the nurse include when discussing alternatives to pain medication with this client? 1. Emphasize that no medication will be given. 2. Review that the use of medications allows for rest and less fatigue. 3. Explain that pain relief will allow a more enjoyable birth experience. 4. Summarize how maternal pain and stress can have a more adverse effect on the fetus than would a small amount of analgesia.

Answer: 4 Explanation: 1. It is important to respect the client's wishes when possible. Once the effects are explained, it is still the client's choice whether to receive medication. 2. While pain relief can allow the client to be less fatigued, it might be the view of the nurse but not the client. 3. While pain relief can lead to a more enjoyable experience, it might be the view of the nurse but not the client. 4. The decision not to medicate should be an informed one, and it is possible that the client does not know about the effects pain and stress can have on the fetus. Once the effects are explained, it is still the client's choice whether to receive medication.

15) The nurse is caring for a pregnant teen. What should the nurse do to accurately assess the teen's nutritional intake? 1. Assess laboratory values. 2. Ask about cooking facilities. 3. Observe for clinical signs of malnutrition. 4. Ask to complete a dietary recall to identify eating patterns.

Answer: 4 Explanation: 1. Laboratory values only provide information about the nutritional status of the client. 2. Cooking facilities might not be related to food intake. 3. Clinical signs of malnutrition only provide information about the nutritional status of the client. 4. In assessing the diet of the pregnant adolescent, it is important to consider the eating pattern over time, not simply a single day's intake. Once the pattern is identified, counseling can be directed toward correcting deficiencies.

14) Being aware that several pregnant students have been diagnosed with iron-deficiency anemia, the school nurse plans a class about nutrition for pregnant teens. What should the nurse encourage the teens to consume to increase iron absorption? 1. Milk 2. Green tea 3. Gatorade 4. Orange juice

Answer: 4 Explanation: 1. Milk does not contain vitamin C. 2. Green tea does not contain vitamin C. 3. Gatorade does not contain vitamin C. 4. Vitamin C is found in citrus fruits and juices and is known to enhance the absorption of iron from meat and nonmeat sources.

12) A client in labor is being prepared for epidural anesthesia. What should the nurse expect to perform in order to prevent the most common complication associated with this anesthesia? 1. Observe fetal heart rate variability. 2. Place the client in the semi-Fowler position. 3. Teach the client appropriate breathing techniques. 4. Rapidly infuse 500 to 1000 mL of intravenous fluids.

Answer: 4 Explanation: 1. Monitoring for fetal heart rate variability will not prevent the most common complication, which is maternal hypotension. 2. Placing the client in the semi-Fowler position will not prevent the most common complication, maternal hypotension. 3. Breathing techniques will not prevent the most common complication, maternal hypotension. 4. Administering a fluid bolus prior to an epidural generally prevents maternal hypotension, which is the most common disadvantage of the procedure.

3) After teaching a class about the female reproductive system, the nurse asks the attendees to describe the process of meiosis. Which student response suggests successful comprehension of the material? 1. "At the time of ovulation, the first meiotic division begins." 2. "At the time of puberty, the second meiotic division begins." 3. "Completion of the first meiotic division produces three polar bodies and one primary oocyte." 4. "Completion of the second meiotic division results in formation of three polar bodies and one ovum."

Answer: 4 Explanation: 1. The first meiotic division of oocytes begins before the female fetus is born. 2. The second meiotic division begins at the time of ovulation. 3. Completion of the first meiotic division produces one polar body and one secondary oocyte. 4. Completion of the second meiotic division produces three polar bodies and one ovum. Page Ref: 136

5) Upon arriving at the prenatal clinic in the morning, the nurse receives messages from four clients. Which client complaint should be addressed first? 1. Primipara at 24 weeks' gestation with ankle edema 2. Multipara at 35 weeks' gestation with new onset of hemorrhoids 3. Primipara at 9 weeks' gestation with increased fatigue and nocturia 4. Multipara at 30 weeks' gestation with vaginal bleeding after performing yoga

Answer: 4 Explanation: 1. Particularly during the second and third trimesters, ankle edema is a common pregnancy-related discomfort. 2. Hemorrhoids are a common pregnancy-related discomfort, especially during the second and third trimesters. 3. Fatigue and increased urination at night is a common pregnancy-related discomfort during the first trimester. 4. Vaginal bleeding after yoga is a warning sign that should be immediately reported to the healthcare provider; this client is the highest priority for care. Page Ref: 211

9) A pregnant client who is a lacto-ovo vegetarian asks for help planning a diet that includes adequate protein intake. What instruction should the nurse give? 1. "To improve protein absorption, avoid simultaneous intake of animal protein and plant protein." 2. "Following a lacto-ovo vegetarian diet will require you to take a daily supplement of vitamin B12." 3. "Because you don't eat meat, eggs, or dairy products, it's important to eat adequate plant-based proteins." 4. "In addition to eggs and dairy products, beans, peanut butter, and soy milk can be effective sources of plant-based proteins."

Answer: 4 Explanation: 1. Plant protein quality can be improved if it is consumed with certain animal proteins. 2. Vegan diets, in which no animal products are consumed, often require daily supplementation of vitamin B12. 3. While lacto-ovo vegetarians do not eat meat, they do eat eggs, milk, and dairy products. 4. A diet that includes plant proteins, such as beans and rice, peanut butter on whole-grain bread, and whole-grain cereal with soy milk, helps ensure the expectant mother obtains all the essential amino acids.

5) The nurse is working with a group of pregnant teens. Which statement indicates that teaching has been successful? 1. "My baby could come late because I am a teenager." 2. "Because I am young, I have a low risk for preeclampsia." 3. "I am more likely to use birth control after I have this baby." 4. "Pregnant teens are more likely to quit school prior to graduation." `

Answer: 4 Explanation: 1. Postdates are not a risk for adolescent pregnancy; preterm birth and small-for-gestational-age infants are risks for pregnant teens. 2. Preeclampsia is more likely to occur in teen mothers than in mothers older than age 19. 3. Clients who give birth the first time as adolescents are more likely to have their next child during adolescence as well. Birth control use is not higher among teen parents. 4. Clients who give birth the first time as adolescents are more likely to have lower educational levels, including a higher rate of dropping out of high school and not attending college or vocational training. Page Ref: 238

7) Which statement, if made by a pregnant adolescent, indicates that she understands her increased risk of physiologic complications during pregnancy? 1. "Smoking and using crack cocaine won't harm my baby." 2. "My anemia and eating mostly fast food are not important." 3. "It's no big deal that I started prenatal care in my seventh month." 4. "I need to take good care of myself so my baby doesn't come early."

Answer: 4 Explanation: 1. Pregnant adolescents are at great risk for complications such as pregnancy-induced hypertension, anemia, preterm birth, low-birth-weight infants, fetal harm from cigarette smoking, alcohol consumption, or the use of street drugs. 2. Pregnant adolescents are at great risk for complications such as anemia. 3. Early and regular prenatal care is the best intervention to prevent complications or to detect them early, to minimize the harm to both the teen and her fetus. 4. Early and regular prenatal care is the best intervention to prevent complications or to detect them early, to minimize the harm to both the teen and her fetus. Page Ref: 241

16) A pregnant couple would like their 5-year-old to attend the birth. Which should the nurse say in response to this couple's plan? 1. "Bring some toys to keep your child occupied." 2. "Children under 12 are not allowed to be present at the birth." 3. "You should let your child stay home because you will be focusing on the birth." 4. "You should bring someone who will only tend to any specific needs of your child."

Answer: 4 Explanation: 1. Preparing the child on what to expect is beneficial. Toys will not sustain a 5-year-old's attention for an extended time period. 2. Children are allowed to be present at births. 3. A sibling should have his own support person whose primary responsibility is to take care of the child's needs so that the child will have support if anxiety develops over the birth process, and the mother can concentrate on the labor and birth. 4. A sibling should have his own support person whose primary responsibility is to take care of the child's needs so that the child will have support if anxiety develops over the birth process, and the mother can concentrate on the labor and birth. Page Ref: 202

1) A client in labor who rates pain as 9 on a scale from 1 to 10 requests pain medication after refusing epidural anesthesia. What action should the nurse take prior to administering butorphanol tartrate (Stadol) as prescribed? 1. Offer epidural anesthesia again. 2. Administer oxygen via face mask at 6 to 10 L per minute. 3. Obtain maternal vital signs and assess the fetal heart rate (FHR). 4. Instruct on the actions and contraindications associated with the medication

Answer: 4 Explanation: 1. The client has refused epidural anesthesia but is authorized to receive butorphanol tartrate. 2. Routine oxygen administration is not indicated for administration of butorphanol tartrate to an asymptomatic client in labor. 3. Prior to obtaining maternal vital signs and assessing FHR, the nurse should advise the client as to the actions and contraindications associated with butorphanol tartrate. 4. Prior to administering the medication, the nurse must explain the pharmacologic effects of

A client in labor is a Lebanese immigrant, and she explains that in the Muslim faith, the baby's name is selected after delivery, as it is God's will whether or not the baby will be born. Which nursing response is appropriate? 1. "Are you afraid your baby will not live?" 2. "We have a very low rate of complications at this facility." 3. "In the United States, you can feel free to choose your baby's name prior to the delivery." 4. "Thank you for explaining that to me. By sharing your cultural beliefs with me, you are helping me to provide you with the best possible care."

Answer: 4 Explanation: 1. The client is describing the application of a culturally based belief, not a fear-based concern. 2. The client is describing the application of a culturally based belief, not a concern about complications. 3. It is neither realistic nor appropriate to assume that people of another culture will automatically abandon their ways and adopt the practices of a dominant culture. 4. The identification of cultural values is useful in planning and providing culturally sensitive care.

11) Which statement best indicates that the client understands the differences in the follicular and luteal phases of the ovarian cycle? 1. "My period will be every 28 days." 2. "The follicular phase is when the egg is fertilized." 3. "The follicular phase is the second half of my cycle." 4. "The first part of my period might vary in length, but not the second."

Answer: 4 Explanation: 1. The follicular phase can vary, resulting in cycle length other than 28 days. 2. The follicular phase comprises days 1 to 14 of the menstrual cycle, not when the egg is fertilized. 3. The luteal phase is the second half of the cycle. 4. For a female with a 28-day cycle, the follicular phase comprises days 1 to 14 of the menstrual cycle, and the luteal phase comprises days 15 to 28. The luteal phase does not vary.

14) In preparation for teaching a women's community center class about physiologic changes during menopause, the nurse is preparing a handout for students. Which information should the nurse include in this teaching? 1. Due to changes in estrogen levels, the labia minora increase in size after menopause. 2. The ovaries remain small after puberty, but they increase in size following menopause. 3. After menopause, the endometrium continues to undergo monthly degeneration and renewal. 4. Ovarian secretion of estrogen decreases between the ages of 45 and 55, after which point ovulatory activity ceases.

Answer: 4 Explanation: 1. The labia minora decrease in size after menopause because of changes in estrogen levels. 2. The ovaries of girls are small, but they become larger after puberty and then decrease in size following menopause. 3. From menarche to menopause, the endometrium undergoes monthly degeneration and renewal in the absence of pregnancy. 4. Between the ages of 45 and 55, a woman's ovaries secrete decreasing amounts of estrogen. Eventually, ovulatory activity ceases and menopause occurs.

10) Which statement by a parent of a pregnant, unmarried 15-year-old is expected? 1. "I'm not going to get involved. She understands how her health insurance works." 2. "An abortion is the best choice for her. She can deal with our Catholic priest later." 3. "We're very happy for her. It will be easier to focus on education with a new baby." 4. "Her father told her to stop dating that boy. Now look at the trouble she's gotten into."

Answer: 4 Explanation: 1. The parent of a pregnant teen is usually the support person and helps the teen understand how to access prenatal care. It is unlikely that a 15-year-old would understand health insurance. 2. This statement indicates that the parent is not going to discuss the pregnancy with the teen but might be forcing the teen into abortion. Because religious tradition impacts views on abortion, and Catholicism disapproves of abortion, the teen might not accept abortion as an option. 3. Most parents accept the pregnancy but are not excited when their 15-year-old is pregnant. And education is harder when child care is involved. Teens that give birth are less likely to complete their education. 4. This statement indicates anger and dismay, which are expected when a parent finds out about a teen daughter's pregnancy. Page Ref: 239

3) A multigravida gave birth to an 18-week fetus last week. During her follow-up she sees that it is documented in her medical record that she had one abortion and becomes upset over the use of this word. How can the nurse best explain this terminology to the client? 1. "Abortion is what we call all babies who are stillborn." 2. "Abortion is the word we use when someone has miscarried." 3. "Abortion is how we label pregnancies that end in the second trimester." 4. "Abortion is the medical term for all pregnancies that end before 28 weeks."

Answer: 4 Explanation: 1. Third-trimester losses are considered fetal death in utero, and the term abortion is not used. 2. Abortions are fetal losses prior to the onset of the third trimester and include elective induced (medical or surgical) abortions, ectopic pregnancies, and spontaneous abortions or miscarriages. 3. Abortions are fetal losses prior to the onset of the third trimester and include elective induced (medical or surgical) abortions, ectopic pregnancies, and spontaneous abortions or miscarriages. 4. Abortions are fetal losses prior to the onset of the third trimester and include elective induced (medical or surgical) abortions, ectopic pregnancies, and spontaneous abortions or miscarriages. Page Ref: 171

11) A pregnant client who swims 3 to 5 times per week asks the nurse if she should stop this activity. What is the appropriate nursing response? 1. "You should discontinue your exercise program immediately." 2. "You should increase the number of times you swim per week." 3. "You should decrease the number of times you swim per week." 4. "You should continue your exercise program because it would be beneficial."

Answer: 4 Explanation: 1. Thirty minutes of moderate-intensity exercise daily is recommended for pregnant women, but even mild exercise is helpful. There is no reason for the exercise to be discontinued. 2. Thirty minutes of moderate-intensity exercise daily is recommended for pregnant women. There is no reason for the exercise to be increased. 3. Thirty minutes of moderate-intensity exercise daily is recommended for pregnant women. There is no reason for the exercise to be decreased. 4. Thirty minutes of moderate-intensity exercise daily is recommended for pregnant women, but even mild exercise is helpful. Women who exercise regularly have better muscle tone, self-image, bowel function, energy levels, sleep, and postpartum recovery than do those who are sedentary. Page Ref: 211

10) The nurse is seeing prenatal clients in the clinic. Which client is exhibiting expected findings? 1. Primipara at 26 weeks with fundal height of 30 cm 2. Multipara at 12 weeks who reports bright red vaginal bleeding 3. Multipara at 22 weeks who reports no fetal movement felt yet 4. Primipara at 12 weeks with fetal heart tones heard by Doppler fetoscope

Answer: 4 Explanation: 1. This is an abnormal finding. Beginning in the second trimester, the fundal height should correlate with weeks of gestation; thus, at 26 weeks' gestation, the fundal height should be about 26 cm. 2. This is an abnormal finding. Bright red bleeding during pregnancy is never expected. 3. This is an abnormal finding. Fetal movement should be felt by 20 weeks. 4. This is an expected finding because fetal heart tones should be heard by 12 weeks using an ultrasonic Doppler fetoscope. Page Ref: 180

8) The nurse is planning an educational session for pregnant clients who are vegans. What information should the nurse include? 1. Vegan diets are high in iron. 2. Rice contains a high level of vitamin B12. 3. Soy is not a good source of protein for vegans. 4. Eating beans and rice provides complete protein needs.

Answer: 4 Explanation: 1. Vegan diets are low in iron, and pregnant vegans often experience anemia. 2. Rice does not contain vitamin B12. 3. Soy is a very good source of protein and calcium and is safe during pregnancy. 4. Complete proteins can be obtained by eating different types of plant-based proteins such as beans and rice.

17) A pregnant client is having pelvimetry measurements made. Which diagram should the nurse use to demonstrate the technique for determining the anteroposterior diameter?

Answer: 4 Explanation: 1. This diagram demonstrates the use of the McDonald method to assess fundal height. 2. This diagram demonstrates the use of a closed fist to measure the outlet. 3. This diagram demonstrates how the diagonal conjugate is estimated, which extends from the lower border of the symphysis pubis to the sacral promontory. 4. This diagram demonstrates how the anteroposterior diameter of the outlet is estimated, which extends from the lower border of the symphysis pubis to the tip of the sacrum. Page Ref: 187

4) The charge nurse is reviewing the plans of care for four clients in labor. Which care plan requires additional information before implementing? 1. Administration of a spinal anesthetic to a client who is scheduled for a vaginal delivery 2. Administration of a spinal anesthetic to a client with a history of irritable bowel syndrome (IBS) 3. Administration of epidural anesthesia to a client who is in the first stage of labor and has a shellfish allergy 4. Administration of epidural anesthesia to a client with a history of vomiting secondary to hyperemesis gravidarum

Answer: 4 Explanation: 1. Spinal anesthetics may be used to provide anesthesia for cesarean birth and occasionally for vaginal birth. 2. Spinal anesthesia is not contraindicated for irritable bowel syndrome (IBS). 3. A lumbar epidural relieves pain associated with the first and second stages of labor. An allergy to shellfish is not a contraindication to epidural anesthesia. 4. Contraindications to epidural anesthesia include severe hypovolemia of any etiology. The client with hyperemesis gravidarum should be evaluated for severity of dehydration prior to administration of epidural anesthesia.

6) Prior to receiving lumbar epidural anesthesia, in which position should the nurse place the client in labor? 1. Lying prone with a pillow under the chest 2. On the right side in the center of the bed with the back curved 3. On the left side with the bottom leg straight and the top leg slightly flexed 4. Sitting on the edge of the bed with the back slightly curved and feet on a stool

Answer: 4 Explanation: 1. This position is not consistent with access to the epidural spaces. 2. Especially in pregnant women, this position is not ideal for facilitating access to the epidural space. 3. This position is not consistent with access to the epidural spaces. 4. Sitting on the edge of the bed with the back slightly curved and the feet on a stool allows the epidural spaces to be accessed more easily.

7) The nurse learns that a pregnant client's religion is Judaism. Why is this information important? 1. Religious and cultural background can impact what a client eats during pregnancy. 2. Knowing the client's beliefs and behaviors regarding pregnancy is important. 3. Clients sometimes encounter problems in their pregnancies based on what religion they practice. 4. It provides a baseline from which to ask questions about the client's religious and cultural background. Answer: 4

Explanation: 1. Although this is true, much more than diet is impacted by religious and cultural background; values, beliefs, expectations for the birth, and acceptance or refusal of medical treatment are also influenced by religious or cultural background. 2. Not all people interpret or live out their religious or cultural backgrounds the same way. It is imperative to avoid stereotyping clients based on their background. The nurse must use the information on the client's background as an educated starting point on which to base further questions about how this specific client enacts her religious or cultural background. 3. How a client enacts her religion occasionally will cause problems with pregnancy. But the most important reason for asking a client for her religious or cultural background is to have a starting point on which to base further questions on the specifics of how this client is impacted by or enacts her cultural or religious background as a unique individual. 4. This is the best explanation because not all people interpret or live out their religious or cultural backgrounds the same way. It is imperative to avoid stereotyping clients. Thus, the nurse should use the information on the client's background as an educated starting point on which to base further questions about how this specific client enacts her religious or cultural background. Page Ref: 176

2) The nurse has completed a presentation on reproduction. Which participant's statement indicates that teaching has been successful? 1. "Ova separate into two unequally sized cells." 2. "A male is born with all the sperm he will ever produce." 3. "Females create new ova throughout their reproductive life." 4. "Each primary spermatocyte divides into four haploid cells."

Explanation: 1. Each ovum undergoes meiotic division just prior to being released from the graafian follicle. Each cell created by this meiosis has the same number of chromosomes, but the cytoplasm does not split equally. This causes a polar body to be produced along with a secondary oocyte. 2. Males begin spermatogenesis at puberty and continue throughout their life. Each sperm divides into four haploid cells. 3. Females are born with all the ova they will ever produce. The ova begin to be formed in early fetal life. One ovum is released each month during the reproductive life of a female, from menarche to menopause. 4. Primary spermatocytes contain 46 chromosomes (46XY). Each primary spermatocyte undergoes meiotic division into two haploid secondary spermatocytes (22X or 22Y).


Related study sets

Management of Patients with Hematologic Neoplasms

View Set

EC111 Chapter 3 Review Questions

View Set

Module 1: Criminal Offenses in Illinois

View Set

Psy201 Ch 8 INTRODUCTION TO HYPOTHESIS TESTING

View Set

NUR334 Reproduction and GU, ATI Review

View Set